Sie sind auf Seite 1von 41

SOLUTIONS

MCAT
*
Practice Test 8
MCAT
Medical College
Admission Test
www.PrincetonReview.com
AAMC
MCAT Practice Test 8 Solutions 2
MCAT Practice Test 8 SOLUTIONS
Edited, produced, typeset, and illustrated by
Steve Leduc
Director, MCAT Program Development
The Princeton Review
CONTENTS PAGE
Physical Sciences solutions ..................................................... 3
Steve Leduc (Physics)
Steve Leduc, Bethany Blackwell (General Chemistry)
Verbal Reasoning solutions ................................................... 12
Jennifer Wooddell
Biological Sciences solutions ................................................ 33
Judene Wright (Biology)
Bethany Blackwell (Organic Chemistry)
Copyright 2005 by Princeton Review, Inc.
All rights reserved.
*MCAT is a registered trademark of the Association of American Medical Colleges (AAMC).
The Princeton Review is not afliated with Princeton University or AAMC.
This document is for the exclusive use of Princeton Review course students,
and is not legal for resale.
www.PrincetonReview.com
050525
Physical Sciences Solutions 3
PHYSICAL SCIENCES SOLUTIONS
Passage I (Questions 15)
1. A. Since sulfur and uorine are both nonmetals, we would expect a compound of S and F to be covalent (since, as mentioned
in the passage, the vast majority of covalent compounds are comprised exclusively of nonmetallic elements). This eliminates
choices B and D. Furthermore, the passage also tells us that aqueous solutions of covalent compounds do not conduct electricity.
Therefore, the answer is A.
2. A. Metals are found on the left side of the periodic table, nonmetals on the right side.
3. D. The passage states that the vast majority of covalent compounds are comprised exclusively of nonmetallic elements.
Since both hydrogen and bromine are nonmetals, HBr is the only compound in the answer choices that can be the covalent one,
so the answer must be D.
4. B. The binary compound with the most ionic character will be the one whose constituent elements have the greatest electro-
negativity difference. Since Cs is directly below Rb in the periodic table, we expect its electronegativity to be no greater than
that of Rb. So, the electronegativity difference for the elements in CsCl (choice B) will be no less than 3.0 0.8 = 2.2. No pair
of elements appearing in a compound in any of the other three choices has an electronegativity difference as great as this. (For
choice A, KBr, the electronegativity difference is 2.8 0.8 = 2; for choice C, NaI, the difference is 2.5 0.9 = 1.6; and for choice
D, RbBr, the difference is 2.8 0.8 = 2.)
5. C. The passage states that aqueous solutions of ionic compounds conduct electricity. So, the fact that a 1 M solution of
HCl conducts electricity would be consistent with the claim that HCl is ionic.
Passage II (Questions 612)
6. A. The last sentence of the third paragraph of the passage states that B
3
is the vector sum of the magnetic elds of electrons
and other nuclei in the vicinity of the H atom [emphasis added]. Therefore, B
3
is determined by the environment of the nucleus,
not by its (intrinsic) mass, radius, or charge.
7. C. The rst sentence of the last paragraph of the passage tells us that if is antiparallel to B
1
, the H nucleus eventually will
relax . . . and emit energy . . . [emphasis added], so choice C (not D) is correct. Choice A is wrong because were told in the
third paragraph that each H atom has a nonzero (and only nuclei with a net spin of zero have = 0). Choice B is eliminated
because the magnitude of the externally produced (and controlled) static magnetic eld B
1
is not affected by .
8. B. The third paragraph of the passage begins by stating that MRI uses magnetic elds to produce high-contrast images of
human tissue. We are told in the second paragraph that dyes often must be injected in order to enhance contrasts in images.
Clearly, then, dyes may be used (even used often), but they dont have to be used, so choice B is better than C or D. As for choice
A, the passage contains no discussion of toxicity (and this choice begs the question, Less toxic than what?).
9. C. The second paragraph of the passage discusses contrast. We are told that contrast results from differences in attenuation,
and that attenuation coefcients are roughly proportional to the atomic numbers of the elements contained in the tissue. If the
contrast is too low, this means that differences in attenuation are too low, which immediately implies that differences in the
atomic numbers of the elements comprising the target tissue are too low. This is directly stated in choice C.
10. C. The formula for
d
is given in the fourth paragraph of the passage:
d
= 4B
1
/h. If B
1
is nonzero, then the nuclear
magnetic dipole moment, , must be nonzero in order for
d
to be nonzero. So, the question becomes, Which one of the four
nuclei given as choices here has a nonzero ? The magnetic moments of paired nucleons cancel each other out, so any nucleus
with an even number of protons (even Z) and an even number of neutrons (even N) will have = 0. But even if you didnt know
MCAT Practice Test 8 Solutions 4
this fact from nuclear physics, you can still choose the correct answer. All of the choices have an even Z (and an even A, which
implies an even N), except choice C, which has an odd Z. We are told in the passage that each H atom, which we know has an
odd number of protons (namely, 1) has a nonzero . We conclude, then, that nucleus C, the only choice with an odd number of
protons has, like an H atom, a nonzero .
11. A. The formula for
d
is given in the fourth paragraph of the passage:
d
= 4B
1
/h. For a given nucleus, is xed, so
d

varies only with B
1
(all other quantities in the formula are constants). Thus, B
1
is the only quantity that a diagnostician could
alter to adjust
d
of H nuclei.
12. C. As stated in the fourth paragraph of the passage, If the rotational frequency of B
2
equals
d
(a condition called
resonance), the nucleus can become antiparallel to (make a 180 angle with) B
1
.
Passage III (Questions 1317)
13. C. To see the effect of temperature on the reaction rate, we would want all other variables to remain unchanged. Tube 5
begins with 20 mL of Solution A, 20 mL of Solution B, and nothing else. Of the other trials listed in Table 1 of the passage,
only Tubes 1 and 4 have these same starting materials (note that an additional compound was added to Tube 6, which therefore
changed the initial composition in this trial). Since Tube 1 is not listed among the choices, we select Tube 4 (choice C).
14. D. The reactions shown in the passage include I

, S
2
O
3
2
, and S
2
O
8
2
as reactants, so we would expect that adding any of
the compounds listed in choices A, B, or C would affect the results. However, the passage tells us that starch is used only as an
indicator, so it is not an active participant in the reactions under study. Therefore, adding starch to Solution B would most likely
not affect the experimental results.
15. D. Table 1 shows that the trials using Tube 1 and Tube 6 had the same starting conditions: the same volume of Solution A,
the same volume of Solution B, the same volume of water, and the same temperature. However, the time measured for Tube 6
is a full 10 seconds less than the time measured for Tube 1. Since the only difference between Tubes 1 and 6 was the addition
of CuSO
4
to Tube 6, and the result was an marked increase in reaction rate, we conclude that CuSO
4
was a catalyst.
16. B. First, we can eliminate choices C and D, because according to Table 1, the initial volumes of the solutions for the trials
using Tubes 1 and 4 were the same. Choice A is backwards. The passage tells us that the solution turns dark blue when one
of the reactants of Reaction 2, namely S
2
O
3
2
, gets used up. So, if the rate of Reaction 2 were slower in Tube 1 (as claimed in
choice A), then the solution in Tube 1 would have taken longer to turn dark blue. The answer is B. Tube 1 was run at a higher
temperature than Tube 4, so the average kinetic energy of the reactants was greater in Tube 1; this greater kinetic energy led to
the faster reaction rate.
17. B. Reaction 2 proceeds until one of its reactants (S
2
O
3
2
) is used up and the solution turns dark blue. According to Table
1, this reaction proceeds in Tube 6 for 19 seconds. At this point, then, no more of the product S
4
O
6
2
will be formed. So, the
amount of S
4
O
6
2
should rise until t = 19 seconds and remain constant thereafter. This is shown by the graph in choice B.
Passage IV (Questions 1822)
18. B. The last two sentences of the passage tell us that the ratio of d to t
2
increased as the angle of inclination increased.
Therefore, this ratio should have attained its maximum value when the angle of inclination reached its maximum valuethat
is, when the inclined plane was vertical.
19. A. Aristotles theory of natural motion deals with objects moving through the air at a constant velocity, so choice B
is eliminated. A mass on a spring is acted upon by a force outside the object (namely, the force from the spring) and thus,
according to Aristotle, undergoes violent motion. This eliminates choice C. The statement in choice D is also dealt with by
Aristotles theory; in fact, the rst sentence of the passage states that Aristotle believed that the time it takes for an object to
Physical Sciences Solutions 5
fall a given distance is inversely proportional to its weight, which would imply that a heavy body should fall faster than a
lighter one. The answer must be choice A. Aristotles theory of natural motion says that an object in motion is propelled at a
constant velocity by the air.... This theory would therefore not be able to explain an object moving with constant velocity in
the absence of air.
20. C. First, we eliminate choices B and D because theyre not true. The experiments of Galileo that are described in the passage
were used to measure d (distance traveled) and t (the time of travel) and then determine the ratio d/t
2
. It seems easy enough to
measure a distance of 10 m with a reasonable degree of accuracy. However, it would have been far more difcult for Galileo to
measure very short time intervals (less than, say, a couple of seconds) with a comparable degree of accuracy. (A sphere dropped
from a height of 10 m would reach the ground in less than a second and a half.) Furthermore, any error incurred in measuring t
would be amplied because t is squared in the ratio Galileo was calculating. Therefore, the main advantage in using an inclined
plane (whose angle could be made as small as desired, thereby making the spheres travel time as long as desired) is that the
spheres would take longer to reach the bottom. Note that while choice A is generally true, Galileo was not measuring the nal
velocity of the spheres, only the distance traveled and their travel time, so choice C is a better response than A.
21. B. The rst sentence of the passage states that Aristotle believed that the time it takes for an object to fall a given distance
is inversely proportional to its weight. Therefore, since all the objects are dropped from the same height, the object with the
greatest weight (which is one with the greatest mass), Sphere B, would fall to the ground the fastest, so it would have been the
one that achieved the greatest terminal velocity.
22. D. The rst sentence of the passage states that Aristotle believed that the time it takes for an object to fall a given distance
is inversely proportional to its weight. Since weight is directly proportional to mass, it follows that Aristotles theory would
predict that the time it takes for an object to fall a given distance is inversely proportional to its mass. Lets compare the masses
of Objects A and B described in the question. Since mass = density volume, and Object A has twice the density but the
volume of Object B, Object A will have the same mass as Object B. (Note that this eliminates choices B and C.) Because their
masses are the same, these spheres should therefore take the same amount of time (choice D) to fall a given distance, according
to Aristotle. Note that choice A is a trap: Aristotles theory about the time required for an object to fall a given distance depends
on the objects weight, not its density.
Free-Standing Questions (23 through 26)
23. D. The solubility equilibrium of M(OH)
2
is given by M(OH)
2
M
2+
+ 2 OH

, so if [M
2+
] = S, then [OH

] = 2S. The
solubility product is given by K
sp
= [M
2+
][OH

]
2
, which thus becomes S(2S)
2
= S(4S
2
) = 4S
3
.
24. C. Treating the gas as an ideal gas, well invoke the Ideal-Gas law, PV = nRT. Since the quantity of gas remains unchanged,
n is a constant. Therefore, PV T, so if T increases, the product PV increases. Which of the choices correctly describes the
behavior of V if the product PV increases? Only choice C: If PV increases and P is constant, then V must increase. (This also
shows why choice D is wrong.) Choice A is eliminated because if P increases by a greater factor than T increases, then V would
decrease. And choice B is wrong because V cannot be inversely proportional to T if PV T.
25. B. When an object oats, its weight is balanced by the buoyant force. The weight of an object with total volume V is given
by mg, which is
obj
Vg. The buoyant force is given by
uid
V
sub
g, where V
sub
is the volume that is submerged. Therefore, for
an ice cube oating in a soft drink, we have
ice
Vg =
drink
V
sub
g, which, after canceling the gs on both sides, becomes
ice
V =

drink
V
sub
, so V
sub
/V =
ice
/
drink
. Since the gravitational acceleration g cancels out of the equation, it wont matter whether the
astronaut is on the surface of the moon or on the surface of the earth. The fraction of the ice cube thats submerged, V
sub
/V, is

ice
/
drink
, independent of the local value of g.
26. C. The ability of a long glass ber to transmit light over a long distance with a minimum loss of energy is due to the
phenomenon of total internal reection.
MCAT Practice Test 8 Solutions 6
Passage V (Questions 2731)
27. A. The passage tells us about the high degree of solubility of CaCO
3
in acidic solutions. Therefore, if Reaction 1 is
effective at decreasing the hardness of an acidic water sample, we would expect that it increases the pH, to make CaCO
3

less soluble (and, in fact, the passage also states that as a result of Reaction 1, calcium carbonate precipitates). We therefore
eliminate choices B and D. Since water hardness is directly related to the amount of dissolved CaCO
3
, decreasing the hardness
of water means decreasing the solubility of CaCO
3
. This is directly stated in choice A.
28. A. Atmospheric CO
2
reacts with water to produce carbonic acid, H
2
O + CO
2
H
2
CO
3
. This would increase the acidity of
the water (that is, lower the pH), thereby allowing a greater amount of CaCO
3
to dissolve (since the passage mentions the high
degree of solubility of CaCO
3
in acidic solutions). Therefore, choice A is the answer.
29. C. At STP, 1 mole of an ideal gas occupies 22.4 L, so if the evolved CO
2
(g) occupies 11.2 L, we know that this represents
0.5 mole of CO
2
. From Reaction 1, which is balanced, we can see that for each mole of CO
2
(g) produced, 1 mole of CaCO
3
(s)
is produced. Therefore, 0.5 mol of CaCO
3
is formed. Since the molar mass of CaCO
3
is 40 + 12 + 3(16) = 100 g/mol, 0.5 mol
of CaCO
3
has a mass of 50 g.
30. B. The solubility equilibrium here is CaCO
3
(s) Ca
2+
(aq) + CO
3
2
(aq), so K
sp
= [Ca
2+
][CO
3
2
]. If all the carbonate has
dissociated, then [Ca
2+
] = [CO
3
2
] and we can write K
sp
= [Ca
2+
][Ca
2+
] = [Ca
2+
]
2
. Thus, [Ca
2+
] = (K
sp
)
1/2
= (4.8 10
9
)
1/2
M.
31. D. Equilibrium constants, such as K
sp
, can be changed only by a change in temperature, so we eliminate choices A and B.
Thus, the question becomes whether [CO
3
2
] increases or decreases with the addition of excess Ca
2+
. Since the question tells
us that CaCO
3
precipitates, the ions in solution must be combining to form the solid, thereby lowering the concentration of
CO
3
2
.
Passage VI (Questions 3236)
32. D. The frequency of a simple pendulum is given by the formula f = (1/2)(g/L)
1/2
. Since angular frequency (in radians
per second) is related to frequency f (in hertz) by the equation = 2f, the angular frequency of a simple pendulum is given by
= (g/L)
1/2
. Note that choices A and B can be eliminated because the units in both these choices are incorrect: mg/L has units
of N/m or kgs
2
, and (L/g)
1/2
has units of seconds.
33. C. Any angle in degrees can be converted to radians by multiplying by /180. So, an angle of 30 is equivalent to an angle
of 30 /180 (which is /6) radians.
34. B. In Figure 3, Resistors R
2
and R
1
are in series, so they are equivalent to a single 200 + 200 = 400 resistor (which
well call R
21
). Resistors R
3
and R
SG
are also in series and equivalent to a single 400 resistor, which well call R
3SG
. These two
equivalent resistors, R
21
and R
3SG
, are in parallel, so the overall equivalent resistance, R
eq
, is (400 400)/(400 + 400) = 200 .
Since the battery has a voltage of 12 V and the overall resistance of the circuit is 200 , the current from the battery must be
I = V/R
eq
= (12 V)/(200 ) = (60/1000) A = 60 mA.
35. B. The restoring force is the component of the bobs weight, mg, that is directed toward the vertical equilibrium position.
As the following gure shows, this component is mg sin .
Note: The second diagram in Figure 1 in the passage also illustrates this component.
Physical Sciences Solutions 7
36. D. At the molecular level, we can think of the atoms in the string as if they were held together by tiny springs, which can
be stretched or compressed, and thus determine the tension in the string. Therefore, choice D is best.
Passage VII (Questions 3743)
37. A. The buoyant force acting on an object is equal to the weight of the uid it displaces. Since each lead weight is completely
submerged, the volume of water it displaces is equal to its full volume, V = 4 10
6
m
3
. The weight of a volume V of water is
equal to
H
2
O
Vg, so in this case, we nd that the buoyant force is
H
2
O
Vg = (10
3
kg/m
3
)(4 10
6
m
3
)(10 m/s
2
) = 4 10
2
N.
38. D. The results shown in Figure 1 indicate how the temperature of the air inside the balloon varies with the balloons depth.
Since the temperature of the water is kept uniform throughout the tank, knowing the total mass of the water in the tank is
irrelevant.
39. B. As the balloon rises, its depth decreases, and Figure 1 in the passage shows that as the depth decreases, the temperature
also decreases. (You may nd it helpful to place your nger on the curve in Figure 1 at its right-hand end, and move your nger
along the curve to the left. This movement to the left corresponds to the depth decreasingthat is, the balloon rising. Note that
the temperature decreases.) This immediately eliminates choice C, which says the temperature of the gas increases. Also, as the
balloon rises in the tank, the water pressure on it decreases, so the gas in the balloon is not being compressed as it rises, it is
expanding; this eliminates choice D. Now, since the temperature of a gas is directly related to the average kinetic energy of its
molecules, the fact that the temperature decreases immediately implies that the internal energy of the gas decreases also. Choice
B includes both the expansion of the gas and the resulting decrease in its internal energy, and is the best answer.
40. A. The change in temperature of the gas inside the balloon is a direct result of the work done by the gas expanding the
balloon. If the balloon were sufciently inexible, then any expansionand resulting temperature dropwould be far more
difcult to detect.
41. C. The weight of the balloon (not to mention the weight of the piece of lead thats attached to it) is always acting on the
balloon; this eliminates choices B and D, which do not include weight. Since the balloon is submerged in water, buoyancy also
acts on it. Thus, the answer must be C.
42. D. The Continuity Equation tells us that ow speed is greatest where the cross-sectional area of the ow tube is smallest.
The tank itself has an inside diameter of 5 meters (which implies a cross-sectional area of nearly 20 m
2
!), but the drain pipe has
a cross-sectional area of only 0.2 m
2
next to the tank, which tapers to just 0.1 m
2
at the other end. Since the tapered end has the
smallest cross-sectional area, the ow speed here should be the greatest.
43. B. If we consider the air inside the balloon as an ideal gas, then we can apply the Ideal-Gas law, PV = nRT. Since n and T
remain constant here, P is inversely proportional to V. So, if V doubles, then P is reduced to one-half its original value.
Passage VIII (Questions 4448)
44. D. First, we can eliminate choices B and C: oxidation is a loss of electrons, and reduction is a gain. Since it is reduction
that occurs at the cathode, the answer is D.
45. C. Since calcium is an alkaline earth metal in the second column of the periodic table, it will form a +2 ion to achieve a
noble-gas conguration. Ca
2+
has 20 2 = 18 electrons, and only choice C accounts for 18 electrons.
MCAT Practice Test 8 Solutions 8
46. B. The reactions described in Table 1 are displacement/redox reactions between Group I or Group II metals and water.
The intensity of the resulting reaction is directly related to how easily the metal atoms can be oxidizedthat is, to how easily
electrons can be removed from them. Choice A can be eliminated since electronegativity refers to the strength with which an
atom holds on to shared electrons in a bond, and choice C can be eliminated because metals form cations, while electron afnity
refers to the energy associated with forming an anion. Polarizability (choice D) refers to the ability to create a dipole within a
molecule or ion as the electron cloud becomes unevenly distributed; it is not related to the loss of an electron from the reacting
species, as ionization potential is. The best answer is B.
47. A. The reaction of Ca(s) with water is Ca + 2 H
2
O Ca
2+
+ 2 OH

+ H
2
. Because hydrogen gas is produced, we eliminate
choices C and D. To determine the resulting volume of hydrogen gas, we rst note that according to the given balanced reaction,
for each mole of Ca consumed, one mole of H
2
is produced. Since 0.40 g of calcium is equal to 0.01 mole (because the molar
mass of calcium is 40 g/mol), this reaction will produce 0.01 mole of H
2
(g). Now, since the given conditions are very close to
STP, we can say that 1 mole of gas will occupy approximately 22.4 L. Therefore, 0.01 mole of H
2
(g) will occupy approximately
0.224 L = 224 mL, so choice A is best. Note: You could calculate a more precise value of the volume using the Ideal-Gas law
in the form V = nRT/P: with n = 0.01 mol, R = 0.0821 Latm/molK, T = 27C = 300 K, and P = 1 atm, you would nd that
V =
nRT
P
=
(0.01 mol)(0.0821
Latm
molK
)(300 K)
1 atm
= (0.0821)(3) 0.25 L = 250 mL
48. A. Because each of the metals described in the question produces a different color when heated, it follows that each emits
light at specic wavelengths. An element emits light when electrons in higher energy levels drop to lower energy levels.
Since the energy levels are quantized, the photon energies emitted by these transitions are also quantized, so light of only
specic wavelengths is produced. Choice A states this. Choices B and D describe absorption of light of specic wavelengths
(not emission, which is the phenomenon were trying to account for here), and choice C says that electrons are converted into
gases, which is false.
Free-Standing Questions (49 through 53)
49. D. Since Point A is in the liquid phase and Point B is in the gas phase, the arrow from A to B represents a liquid-to-gas
phase change. This is vaporization.
50. D. Choice D describes exactly what makes a material a good insulator/poor conductor: its electrons cannot move easily
from one atom to another.
51. D. Starting with the formula q = mcT, we solve for c and get c = q/(mT). If q = 250 J, m = 0.1 kg, and T = 25C 20C
= 5C, then c = q/(mT) = (250 J)/(0.1 kg 5C) = 500 J/(kgC).
52. B. The resistance of a wire is determined by the formula R = L/A, where is its resistivity, L its length, and A its cross-
sectional area. These are exactly the quantities listed in choice B.
53. B. Most stable means lowest energy. The electron conguration of a nitrogen atom in its ground state is 1s
2
2s
2
2p
3
,
choice B. All the other choices give the conguration of a nitrogen atom in which an electron in a lower-energy orbital has been
promoted to a higher-energy orbital (2s & 2p in choice A, 1s & 2p in C, and 2p & 3s in D). Each of these three congurations
thus describes a nitrogen atom with more energy (and thus lower stability) than the one described in choice B.
Physical Sciences Solutions 9
Passage IX (Questions 5458)
54. B. The passage tells us that 7.15 g of Na
2
CO
3
10H
2
O was dissolved in the initial 50 mL solution. Since the molar mass of
Na
2
CO
3
10H
2
O is given to be 286.14, we can determine that the amount of Na
2
CO
3
10H
2
O dissolved was
7.15 g
286.14 g mol

7
280
=
1
40
mol
Because each formula unit yields 2 Na
+
ions, there were 2(
1
40
) =
1
20
mol of sodium ions in solution. Finally, we use the denition
1 mole 6 10
23
particles to conclude that the number of sodium ions in solution was
1
20
(6 10
23
) = 3 10
22
.
55. C. Sodium ion, Na
+
, is neutral in solution, and carbonate ion, CO
3
2
, is a polyprotic base. Therefore, a solution of Na
2
CO
3

is basic and thus turns red litmus to blue. Since the reaction between carbonate and water is CO
3
2
+ H
2
O HCO
3

+ OH

,
choice C is best.
56. A. The passage tells us that 6.57 g of NiSO
4
6H
2
O was dissolved in the initial 50 mL solution. Since the molar mass of
NiSO
4
6H
2
O is given to be 262.84, we can determine that the amount of NiSO
4
6H
2
O dissolved was
6.57 g
262.84 g mol
=
6.57
262.84

2
2

13
520
=
1
40
mol
Therefore, there is
1
40
mol of Ni
2+
in the solution. Also, from our calculation in Question 54 above, there is also
1
40
mol of
CO
3
2
in the solution. Thus, the maximum amount of NiCO
3
that could form as precipitate is
1
40
mol = 0.025 mol.
57. C. The carbonate ions in solution react with the strong acid to produce carbon dioxide: CO
3
2
+ 2 H
+
CO
2
(g) + H
2
O.
58. C. Electron transitions (caused by the selective absorption of energy from visible light) between d-orbitals in transition-
metal complexes give rise to color. In this case, the nickel(II) sulfate solution absorbs red light, so the solution appears green
(the complementary color of red). Thus, choice C is the answer. The other choices are irrelevant to the production of color.
Passage X (Questions 5965)
59. A. The passage tells us that when a photons E exceeds W [the work function], the photon ejects an electron from the metal.
The kinetic energy, K, of the ejected electron is the difference between E and W. Since the work function of Metal L is 8.0 eV
but the photon has an energy of only 7.0 eV, the photons E does not exceed W, so no electron will be ejected from Metal L,
which we can consider here to be an electron with no kinetic energy. Only choice A lists 0.0 for Metal L.
60. B. In order to increase the current I, we would want to increase the number of electrons ejected from the metal plate reaching
the collector plate (marked B in Figure 2 of the passage) per unit time. If we were to replace the metal plate with one having
a higher work function (choice A), there will either be no electrons ejected from the plate (if the higher work function is greater
than the energy of the incoming photons) or, if electrons are still ejected, their kinetic energy will be decreased, thus decreasing
the number that could reach the collector plate per second. If the number of photons having energies less than W were increased
(choice C), there will certainly be an effect on the number of electrons ejected, because none of these low-energy photons could
cause the ejection of any electrons. Finally, if the average frequency of the photons were decreased (choice D), their average
energy would be decreased also (because photon energy E is directly proportional to photon frequency f, through the equation
E = hf ), and the situation would be the same as the one described earlier for choice A. The answer must be B. If Plate B were
made more positive relative to Plate A, then the negatively charged electrons would be attracted more strongly to Plate B, thus
increasing the current.
61. D. First, we can eliminate choices A and B. If the wavelength is increased, then the energy of the light is reduced (because
E is inversely proportional to , since E = hc/). Thus, the fringes would not become brighter or bluer. The passage tells us that
the brightest fringes occur where wave maxima overlap with other maxima or where minima overlap with other minima. If
is increased, then the maxima and minima move farther apart, so the fringes will move farther apart, too.
MCAT Practice Test 8 Solutions 10
62. C. The passage tells us what causes the fringes to appear on the screen: the overlapping of the two beams from S
1
and S
2
.
In other words, it is the interference of the two beams that creates the fringes; in fact, the passage even calls them interference
fringes. So, if S
1
were covered (choice C), there would be only one beam (the one from S
2
) reaching the screen, so there would
be no interference, and thus no interference fringes. (Diffractive effects are ignored here.) Choice A, B, or D could change the
patterneither causing the fringes to move farther apart (choice A or D) or increasing their contrast (choice B)but none of
these would cause the fringes to disappear.
63. A. As a spherical wave spreads out, its wave front becomes atter. (A portion of a large circle looks straighter than an
equal-angle portion of a small circle.) So, the greater the distance the spherical waves leaving slit S can travel, the atter the
wave fronts will be. Referring to Figure 1, this means that the light entering slits S
1
and S
2
will appear most like a plane wave
if the distance between T
1
and T
2
is large.
64. B. Since Wave A is 180 out of phase with Wave B, when Wave A meets Wave B, they will cancel each other completely.
This leaves just the amplitude of Wave C, which is E.
65. D. First, eliminate choice A: The work function of a metal is an intrinsic characteristic of the metal and is not affected by the
light we choose to shine on it. Choice B is also wrong: The intensity of a beam of light depends on the number of photons, not
on their frequency (or energy)for example, a beam of red light will be more intense than a beam of blue light if the red beam
contains more photons. We also reject choice C, because evidently light waves can carry sufcient energy to eject an electron,
since the photoelectric effect is a real and common phenomenon. The answer is D. If a single photon, which is the least intense
any actual beam of light can be, can cause the ejection of an electron, then this would contradict the wave theory that says
only light of sufcient intensity must be used.
Passage XI (Questions 6671)
66. C. The passage tells us that the ionic compounds [one of which was contained in Solution A] were completely soluble in
water. The choices for this question match the anions listed in Table 1, and of these, only F

gave no precipitate with Ag


+
.
67. D. The passage is not needed to answer this question. When the reaction quotient Q for the dissolution of a compound into
its ionic components is greater than the compounds tabulated solubility product (its K
sp
value), a precipitate forms.
68. A. A reaction in which cations and anions in solution exchange partners and form a precipitate is an example of a meta-
thesis reaction (choice A). Alternatively, you can eliminate the other three choices, none of which applies to the precipitation
reactions in Table 1.
69. B. From Table 1, we see that while the cation Ag
+
caused the formation of three precipitates, and Cu
2+
and Fe
3+
each formed
two, Ca
2+
caused the formation of only one precipitate. Therefore, the cation that allowed for the greatest number of soluble
compounds was Ca
2+
.
70. A. Because BaSO
4
has the lowest K
sp
value, it will form the fewest Ba
2+
ions in aqueous solution, and thus be the least
toxic of the four choices.
71. B. A solution of FeCl
3
contains Fe
3+
and Cl

ions, and a solution of FeF


3
contains Fe
3+
and F

ions. According to Table 1,


the addition of Ca
2+
(choice B) would react with F

, causing a white precipitate to form in the FeF


3
solution, but not react with
Cl

in the FeCl
3
solution. This would certainly allow a researcher to distinguish the two solutions, so choice B is the answer.
Adding CrO
4
2
or S
2
(choice A or C) would, according to the information provided in Table 1, cause brown precipitate to form
in both solutions, while adding Cu
2+
(choice D) would cause no reaction in either solution. Therefore, none of these would be
useful in differentiating between the solutions.
Physical Sciences Solutions 11
Free-Standing Questions (72 through 77)
72. B. The terminal voltage of the battery is its full electromotive force minus the voltage drop across its internal resistance.
Since the current is i and the internal resistance is r, the voltage drop across the internal resistance is ir, so the terminal voltage
of the battery is reduced to ir.
73. B. Because the medium has a higher refractive index, n, than that of the air, the beam will bend toward the normal as it
enters the medium from the air. Therefore, > , which eliminates choices A and C. Since the angle of incidence for the beam
leaving the medium is , the angle of refraction here must be .

air
medium
(n)
sin = n sin n sin = sin
angle of
incidence
angle of
refraction
angle of
incidence
angle of
refraction
Law of Reflection
(That is, the sizes of the angles made with the normal when the beam goes from medium-to-air are the same as when the beam
went from air-to-medium. Only the terms incidence and refraction would be interchanged.) Thus, = .
74. D. We can eliminate choice A since it would be statistically impossible for the molecules of one gas to transfer their excess
kinetic energy to the molecules of another gas if the two gases are in equilibrium. Choice B is wrong since in the kinetic theory
of gases, collisions are elastic, so a molecule colliding with a wall of the container will have the same kinetic energy after the
collision as before. Choice C is wrong because we know from the question that the molecules in the two phases have the same
average kinetic energy; choice C claims that the vapor molecules have a higher kinetic energy. Alternatively, the potential
energy of a substance increases as the phase changes from liquid to gas. Thus, choice C can be eliminated because it says that
the vapor has a lower potential energy. The answer must be D. Molecules that escape the surface of the liquid must expend
energy to overcome the intermolecular forces that held them to the liquid, so the higher kinetic energy these molecules had
before vaporizing is reduced by this expenditure.
75. A. The speed of a wave is determined by the medium it travels through, not by the waves intensity, frequency, or wavelength.
Since the wave is traveling through the same medium (namely, air) both before and after reection, its speed will remain the
same, too; the answer must be A. Choices C and D could be eliminated because of the Doppler Effect: Since the reector is
moving toward the source, we know that the wave after reection will have a higher frequency and a shorter wavelength than
the wave before reection. As for choice B, intensity, the fact that the question describes the reector as imperfect tells us that
the intensity after reection will be less than before.
76. D. The formula for the Doppler shift, f = f (v v
D
)/(v v
S
), involves only the speed of sound in the medium (v), the speeds
of the source and observer (v
S
and v
D
), and the emitted and observed frequencies ( f and f ). It does not involve the distance
between source and observer.
77. D. The impulsemomentum theorem tells us that the change in the momentum of an object is equal to the impulse applied
to it, so the answer is D. Also, since the units of a change in momentum are the same as the units of momentum itself (namely,
kgm/s), a quantity thats equal to a change in momentum must have these units, too. But none of the quantities in A, B, or C
has these units, thus eliminating all three of these choices. (Choice A is a trap. An alternative form of Newtons Second law,
F = p/t, says that force is equal to the rate of change of momentum, not merely a change in momentum.)
MCAT Practice Test 8 Solutions 12
VERBAL REASONING SOLUTIONS
Passage I (Questions 7883)
78. C
A: First and most importantly, the passage never suggests that the U.S. has a low literacy ratethis is outside knowledge.
Second, even if the author did tell us that the U.S. had a low literacy rate, that would not prove that the introduction of an
alphabet changes cognitive habits.
B: There is no research or data cited in the passage to support this claim; it is simply asserted in lines 5657.
C: Yes. There is no reason, based on the passage, to believe the assertion to be false; therefore, it perhaps is true.
However, there is no evidence or reasoning backing up the assertion; it is stated in lines 5657 with no support. The
other items on the list (social relations, notions of community, history, and religion [lines 5759]) dont constitute
separate evidence showing that introducing an alphabet changes cognitive habits.
D: While the author does assert that television viewing is pervasive (lines 2729), this is not inconsistent with the claim that
an alphabet changes cognitive habits. Nothing in the passage indicates that the changes brought about by introduction of
an alphabet are mutually exclusive with watching lots of television.
79. A
Note: The correct answer will be the choice that is most inconsistent with the claim that By ushering in the Age of Television,
America has given the world the clearest available glimpse of the Huxleyan future (lines 2931). Part of this claim is the
argument that people are unaware of the changes brought about by television in particular and technology in general; that is,
they did not know what they were laughing about and why they had stopped thinking (lines 7475), because the Huxleyan
world is difcult to recognize (lines 3538).
A: Yes. This choice weakens the authors argument by indicating that people are in fact aware of how television and
the media affect their lives, and that they can consciously evaluate the content and quality of what they see and hear.
That is, they are able to understand the politics and epistemology of the media (lines 7071).
B: This choice suggests neither that television has no impact (if anything, it suggests that it may have an effect) nor that people
are aware of its effects. This choice is consistent, not inconsistent, with the authors suggestion that people [have] become
an audience and their public business [has become] a vaudeville act (lines 1819); we watch and are perhaps entertained,
but we dont intelligently participate.
C: This choice, like Choice B, is not inconsistent with the authors description of a Huxleyan world as one in which we watch
public business as a form of entertainment (line 19) without really understanding what we are watching and why (lines
7175).
D: This choice has no direct relevance to the authors argument in the passage. The authors claim that Huxleys Brave New
World describes our society better than Orwells 1984 (lines 2224) does not rest on a claim that more people have read
Huxleys work.
80. C
Note: Notice the similarity between choices A, B, and D; all three indicate that people would accept and respond to Huxleys
warning. This is inconsistent with the authors argument in lines 3244. In this case, the correct answer is the one that is not
like the others.
A: The author states that those who voice Huxleys warning must do so at a near hysterical pitchbecause what they
want others to see appears benign, when it is not invisible altogether (lines 3237). (This is in contrast to an Orwellian
or prison-like world, which is much easier to recognize, and to oppose [lines 3738].) Yet because they must raise
their voices to such an extreme level in order to be heard, they may be written off as everything from wimps to public
nuisances to Jeremiahs (lines 3235). The suggestion that people would agree with the warning and act accordingly is
inconsistent with these statements in paragraph 4.
B: See the explanation for Choice A. The passage suggests that people are more likely to ignore or discount such a warning.
C: Yes. The author states that because the aspects of a Huxleyan world are so difcult to recognize and resist (lines
3538), people who speak about this matter must often raise their voices to a near-hysterical pitch, inviting the
charge that they are everything from wimps to public nuisances to Jeremiahs (lines 3235). That is, they will likely
be written off as incorrect and/or too extreme.
D: See the explanation for Choice A. The author suggests instead that people are unlikely to be receptive, in part because the
problematic aspect of Huxleys world appears benign, when it is not invisible altogether (lines 3637).
13 Verbal Reasoning Solutions
81. D
A: The author suggests that people are unaware that public culture has become trivialized (lines 1821 and 7175), not that
they actively prefer what is most trivial. If the public were able to distinguish between which candidates were most and
least trivial, it would be inconsistent with the authors suggestion that they did not know what they were laughing about
and why they had stopped thinking (lines 7475).
B: As in Choice A, this answer suggests a level of awareness on the part of the public that is inconsistent with the authors
description. Furthermore, this choice suggests that the public would resist the trivialization of public life. However, the
author argues that these changes, unlike the creation of an Orwellian world, are likely either to go unnoticed or to appear
benign, and so are much harder to resist and oppose (lines 3738).
C: See the explanation for Choice B. The author suggests that the public would not even notice that the candidates and
discussion were trivial, and that if this were noticed it would likely not be seen as a problem (lines 3544). While an
oppressive Orwellian world is easy to recognize and resist, people are unaware that the changes described by Huxley are
occurring (lines 3538) or that they are potentially disastrous (lines 2021 and 6869).
D: Yes. The author describes Huxleys world as one in which public business becomes a vaudeville act (line 19) and
serious discourse dissolves into giggles (lines 4243). A campaign reduced to trivialities would t this description.
The author also argues that although these changes brought about by television can lead to disaster (lines 2021 and
68-69), the public either doesnt perceive that things have changed or doesnt perceive the changes as dangerous
(lines 3538). Therefore, it is likely that the public would not notice the triviality of the campaign.
82. D
Item I: False. The author suggests just the opposite. The passage indicates that because of television, public business
has become a vaudeville act (line 19), serious discourse dissolves into giggles (lines 4243) and our culture is
being drained by laughter (lines 4344).
Item II: True. In paragraph 3 the author identies technology, but more specically television, as the causal factor in
the creation of a Huxleyan world (lines 2731). The author goes on to say that television imposes a way of
life, a set of relations among people and ideas (lines 4748). The passage also draws an analogy between an
alphabet, movable type, and speed-of-light transmission of images such that when introduced into a society,
all three change its cognitive habits, its social relations, its notions of community, history, and religion, and
that speed-of-light transmission of images (television) does so to an extreme degree (lines 5661).
Item III: True. The author argues that technology is not neutral, and that to make the assumption that technology
is always a friend to culture isstupidity plain and simple (lines 5456). The author also argues that the
Age of Television (technology) has brought us close to realizing a Huxleyan future (lines 2931) and that
[Huxley] believedthat we are in a race between education and disaster (lines 6869). The purpose of the
passage as a whole is to warn us that Huxleys vision of spiritual devastation (lines 1314) and culture
death (line 20) is in danger of being made real through the inuence of technology, in particular television.
83. A
A: Yes. If the presidential election were devoid of serious discourse (line 42) and consisted merely of a personality
contest, this would support the authors claim that we are approaching a Huxleyan world in which public business
has become a vaudeville act (line 19), functioning only as a source of entertainment for an unthinking population
(line 75).
B: This choice (no one) is too extreme. The passage describes those who speak about this matter (lines 3237); some
people are in fact warning the public about a possible Huxleyan future (lines 3031).
C: The contestants in a personality contest do not watch the audience; the audience (the people) watch the politicians (Big
Brother) in a Huxleyan world [lines 1618]). This study offers no support for Orwells prophecies.
D: This is the right answer to the wrong question. The author does refer to a belief in the inevitability of progress (lines
6667) as contributing to the power of the ideology of technology. However, the study cited in the question stem does not
offer evidence that people hold this belief.
MCAT Practice Test 8 Solutions 14
Passage II (Questions 8493)
84. C
A: This choice is inconsistent with the authors statement that useful as the experience of literary creation is, the task of
the student is completely distinct. The student must translate the experience of literature into intellectual terms (lines
1114). This statement is intended to refute the argument, described in the rst paragraph, that one cannot understand
literature unless one writes it (lines 67).
B: This choice is too narrow to be the main idea. While the author would agree that it is true (lines 4248), the point of the
passage as a whole is to argue that the methodologies of literary study are to a signicant degree distinct.
C: Yes. The author describes similarities and overlap between the two (paragraph 3) in order to go on to make the
central argument that literature cannot be studied only through using scientic methodologies (lines 4050 and
6063).
D: This choice is too narrow to be the main idea. The author makes this claim in the last paragraph (lines 5360) only in
service of the larger point that the achievements [of the humanistic disciplines] are nevertheless real and permanent
(lines 5659), and that they represent valid methods of knowing (lines 5356).
85. A
Item I: True. The author states that The student must translate the experience of literature into intellectual terms
(lines 1214), and that The problem is one of how intellectually to deal with art, and with literary art
specically (lines 2122).
Item II: False. The passage argues that literary study must nd a way of dealing with irrational or unrational subject matter
in a rational way (lines 1217) but does not suggest that this requires sorting out the rational from the irrational
elements within a literary work.
Item III: False. In the rst paragraph the author argues that literature as art and literary study are two distinct enterprises;
One is creative, an art; the other, if not precisely a science, is a species of knowledge or of learning (lines 14).
While it has been argued that one cannot understand literature unless one writes it (lines 67), the author disagrees
with this claim that the two must be integrated. Finally, the idea that one must study literature in order to create
literature is never mentioned in the passage.
86. C
A: The author mentions art history (the historian of painting) and musicology in line 19 in order to draw an analogy with
literary study. The point of the analogy is that in all of these elds it is necessary to study sometimes irrational or unrational
subject matter in a rational, intellectual way (lines 1220). Later in the passage, the author argues that strict scientic
methodology is insufcient for the study of literature (lines 4042, 4750, and 6063). Thus, the passage indicates that
art history and musicology, like literature, should not be approached with a strict scientic methodology. This choice
contradicts the passage.
B: The passage argues that one does not need to be a writer in order to study literature (lines 112). Since the author draws an
analogy between literary study on one hand, and art history and musicology on the other (lines 1719), we can infer that
one could be an art historian or musicologist without practicing the art form itself. This choice contradicts the passage.
C: Yes. In the second paragraph, the author rst states that a student of literature must translate the experience of
literature into intellectual terms, assimilate it to a coherent scheme which must be rational if it is to be knowledge
(lines 1215). This requires dealing rationally with irrational or unrational subject matter, and in this way the
student of literature will not be therefore in any other position than the historian of painting or the musicologist
(lines 1719). In the beginning of the third paragraph, the author states that The problem is one of how intellectually
to deal with art, and with literary art specically. Thus the author draws an analogy between literary study, art
history, and musicology, and suggests that all three must be approached intellectually, despite their irrational
components.
D: This choice contradicts the passage. The author draws an analogy between literary study, art history, and musicology on the
basis that they all require dealing with sometimes irrational or unrational elements in a coherent, rational, and intellectual
way (lines 1223). See also the explanation for Choice C, and note that Choice C and Choice D are essentially opposites
of each other.
15 Verbal Reasoning Solutions
87. D
A: This would involve applying the second of the three ways described by the author of transferring the methods of the
natural sciences to the study of literature. That is, it would entail [imitating] the methods of natural science through the
study of causal antecedents and origins. Scientic causality is used to explain literary phenomena by the assignment
of determining causes to economic, social, and political conditions (lines 3035). However, the author goes on to argue
that the transfer of natural science methodology to the study of literature has not fullled the expectations with which it
was made originally (lines 4142). Therefore, the author would not identify this choice as the most important task to be
performed.
B: As in Choices A and C, this choice describes applying methods transferred from the natural sciences, a transfer which the
author indicates is not fully adequate to the purposes of literary study (lines 4042). This choice involves the introduction
of the quantitative methods appropriately used in some sciencesi.e., statistics, charts, and graphs (lines 3537), in this
case statistics.
C: As in Choices A and B, this choice describes the application of methods transferred from the natural science; the author
states that this transfer has not fullled the expectations with which it was made originally (lines 4142). By tracing prior
inuences and the inuence of the story on later works, the scholar would be applying the third scientic method described
in paragraph 2, that is, the attempt to use biological concepts in the tracing of the evolution of literature (lines 3839).
D: Yes. First, this is the only choice that is not tainted by some association with the scientic methods discredited by the
author of the passage (paragraph 2 and lines 4043). Second, by isolating the story elements that explain the storys
popularity, the scholar would be translat[ing] the experience of literature into intellectual terms (the experience
of the story would relate to its popularity) and applying to it a coherent and rational scheme (lines 1215) and
analysis (line 45) by isolating certain story elements.
88. A
A: Yes. This choice paraphrases the passage in lines 4247. In this section, the author is listing the methods which are
common to all types of systematic knowledge, including literary study.
B: While these methods (common to both science and literary study) are described as appropriate in lines 4247, the
second part of this choice is incorrect. The author argues that the methods of science, including objectivity and quantitative
methods (lines 2742) are insufcient, and that the other solution commends itself: Literary scholarship has its own valid
methods, which are not always those of the natural sciences (lines 4749).
C: First, the author describes these methods as appropriate (while not sufcient by themselves) for the study of literature (lines
4247). Second, while the author does suggest that literature includes irrational aspects, the author argues that students of
literature are therefore called upon to nd a rational, coherent, and systematic scheme by which irrationality can be dealt
with intellectually (lines 1222).
D: While the author would agree that the humanities and the natural sciences have, to some degree, different aims and
purposes, induction and deduction etc. are listed as methods common to both science and literary study (lines 4247).
Therefore, their use in the study of literature would be appropriate.
89. D
A: The author argues that creating literature and studying it are two distinct activities (lines 12). While investigating the
process of literary creation could potentially provide support for this distinction, the passage as a whole is about the study
of literature and how this study requires methods that are in part different than scientic methods. Therefore, the kind of
investigation described in this answer choice would not provide the most convincing evidence for passage assertions.
B: This choice is inconsistent with the authors assertions that Literary scholarship has its own valid methods, which are
not always those of the natural sciences but are nevertheless intellectual methods (lines 4850), and that there is this
difference between the methods and aims of the natural sciences and the humanities (lines 6163). See also the explanation
for Choice C.
C: See the explanation for Choice B. The author asserts that, while there is some overlap (lines 4247), there are methods to
literary scholarship that are distinct from scientic methodology. Applying a particular scientic technique to a literary
work as well as to a scientic problem would undermine, not support this assertion.
D: Yes. The author argues that Literary scholarship has its own valid methods, which are not always those of the
natural sciences but are nevertheless intellectual methods (lines 4850). This is the main idea of the passage as a
whole (see also paragraph 5). By comparing a literary to a scientic analysis of a literary work, the author could
demonstrate the differences between the two analyses. This comparison could also demonstrate what might be
learned from a literary analysis that cannot be learned from a purely scientic approach. Thus, out of these four
answer choices, choice D could provide the most convincing evidence in support of the passages main assertions.
MCAT Practice Test 8 Solutions 16
90. B
A: While dening these terms might clarify the meanings of the terms themselves, it would not go very far (and not as far as
Choice B) in clarifying what they mean in the specic context of the statement cited in the question stem.
B: Yes. An example of an unrational statement in a literary work would clarify not only what the author means
by unrational, but also what the author means by saying that literature itself contains unrational elements.
Furthermore, given that unrational is (to most people) a less familiar word than irrational, an example of what
the author means by unrational would go furthest, of these four choices, in clarifying the authors meaning. In
general, the best way (on the MCAT) of clarifying (or strengthening) a statement is to give a specic example in
which that statement is true.
C: The irrationality of the process, and the irrationality or unrationality of certain existing elements in a literary work, are two
separate issues. While an irrational process may lead to the creation of irrational elements, this answer choice would at the
most partially explain why irrational elements were created, not how the elements are irrational or unrational (or what the
author means by unrational elements).
D: As in choice C, this statement might help explain why literature contains irrational elements, but not what the author means
by unrational elements (that is, how these elements are in fact unrational).
91. D
A: This choice is inconsistent with the main idea of the passage. While the author does state that there is a degree of overlap
between scientic methodology and methodology appropriate to other forms of systematic knowledge (lines 4447),
including literary study, the passage as a whole is intended to argue that the study of literature has its own unique methods
(lines 4042, 4750, and 5163). The unique character of literary study, not what it has in common with the natural
sciences, is what makes it important.
B: The author argues that creating literature and studying literature are distinct activities: One is creative, an art; the other
[literary study], if not precisely a science, is a species of knowledge or of learning (lines 24). The author never suggests
that the study of literature leads to increased creativity on the part of the student.
C: This is the right answer to the wrong question. That is, the passage teaches us that truth (truth is mentioned in line 51,
with the implication that truth and knowledge are related) can be discovered through non-scientic means (lines 4850
and 6063), but the author does not suggest that the study of literature is important because literary study itself teaches
us about non-scientic paths to the truth. That is, literary study is important because it offers us this knowledge or truth,
not because it demonstrates the availability of such forms of truth. Compare this choice to Choice D to see the subtle
distinction between the two: teaches that in Choice C versus offers in choice D.
D: Yes. The nal paragraph argues that the humanities had worked out valid methods of knowing before the existence
of the modern scientic method (lines 5356), and that the humanities, including literary study, offers its own forms
of knowledge and truth which cannot be achieved only through the scientic method (lines 4854 and 6063).
Furthermore, in the rst paragraph the author describes literary study as a species of knowledge or of learning
(lines 24). This unique contribution is, according to the passage, what makes the study of literature important; this
is, in fact, the main idea of the passage. Compare this choice to Choice C: in C, literary study is important because it
teaches us about the existence of other (non-scientic) paths to truth or knowledge, while in Choice D literary study
is important because it is that path.
92. B
Item I: False. The author lists analysis as one method which is common to all types of systematic knowledge (lines
4247), including both literary study and science. The success of systematic analysis of text would be an illustration
of the success, not the failure, of at least one method used by the natural sciences.
Item II: True. In lines 4247, the author lists ways in which the natural sciences and literary study overlap. Analysis
is one method common to all types of systematic knowledge. Therefore, a successful systematic analysis of
text would be evidence for this overlap.
Item III: False. The fact that some methods used by the sciences (in this case, systematic analysis) can also be successfully
used to study literature (as stated by the passage itself in lines 4247) is not inconsistent with the claim that other
elds of study have worked out their own valid methods of knowing (lines 5356). The author of the passage
argues that there is some overlap, but that non-scientic elds also have some of their own unique methods of
study.
17 Verbal Reasoning Solutions
93. B
Note: When the question stem states that Elsewhere, the author says, this means that the quote that follows is not from the
passage text. That is, this is a New Information question.
A: The new information in the question draws a distinction between literature on one hand, and sociology, politics, and
philosophy on the other. While Choice A is not inconsistent with that distinction, it goes in the opposite direction by
describing the study of the relationship between literature and social and political conditions.
B: Yes. The new quote in the question stem claims that literature is different from other elds of study (sociology, politics,
and philosophy). This theme is in agreement with the authors claim in the passage that Literary scholarship has
its own valid methods (line 48) and that literary study is a species of knowledge or of learning (lines 34). That
is, both the new quote and this answer choice have the same theme: literature (as well as the study of literature) is
to some degree different.
C: This statement goes too far; it is not an assertion made by the author in the passage. The claim that literature (which is
an artistic pursuit, according to the passage [lines 23]) is distinct and unique doesnt indicate that other pursuits are not
artistic. Nothing in the passage or in the new information in the question indicates that only literature is artistic, nor is it
suggested that being artistic is what distinguishes literature from all other elds.
D: The theme of the new information in the question is the distinction between literature and other elds or forms of
knowledge. The theme of this answer choice is the overlap or similarities between literature and science. As in Choice A,
this answer choice, while not in contradiction with the new information, goes in the opposite direction (similarity rather
than difference).
Passage III (Questions 9499)
94. A
A: Yes. The author discusses the disquieting (line 31) implications of the application of the concepts of natural
selection in paragraph 4. According to James Trel, natural selection as observed on Earthchannels intellect to
predators (lines 3133). Trel also states that Most bright animals are carnivores (lines 3334). Therefore, the
smartest beings may also be aggressive (line 30).
B: This is the right answer to the wrong question. While the passage does make this point in paragraphs 5 and 6, this is the
comforting aspect of natural selection (lines 5457), not the disquieting one.
C: While the passage does suggest that aliens might be more intelligent or knowledgeable than we are (lines 1417 and
6467) and that natural selection may channel intellect to predators (lines 3233), the passage does not go so far as to
suggest that natural selection would itself lead extraterrestrials to be more intelligent than humans. Humans, according
to the passage, also are or have been aggressive predators (lines 3741). Finally, the passage does not indicate that the
prospect of intellectually superior aliens is frightening or disturbing.
D: As in Choice B, this is the right answer to the wrong question. That is, this statement is supported by the passage (lines
5461), but this is the comforting, not the disquieting aspect of natural selection.
95. B
A: If extraterrestrials land unannounced, we would have little choice but to pay heed (lines 13). Speculation about the
nature of aliens would be most crucial if we had a choice of whether or not to communicate (line 36).
B: Yes. The author states that if aliens communicated with us by radio, and therefore we could decide whether or not
to reply, our decision could turn on speculation about what the other beings were like (lines 36).
C: While our speculation would include the issue of whether or not the aliens were likely to be hostile, the passage does not
suggest that we would be most likely to speculate when they were in fact hostile (the question, in that case, would already
be answered). Speculation would be most important in a case where we could decide whether or not to communicate (lines
16).
D: This choice takes words from the passage out of context. The author suggests that if aliens achieved space ight, it could
demonstrate that nuclear knowledge can be acquired without setting in motion Armageddon (lines 1821). However, the
passage does not connect this issue to the issue of the circumstances in which speculation about the nature of extraterrestrials
would be most crucial.
MCAT Practice Test 8 Solutions 18
96. C
A: James Trel suggests that aliens might be aggressive, territorial, and quick to reach for the sword (lines 2631). The
statement that alien civilizations might avoid self-destruction through abhorrent means would support, not counter, Trels
views.
B: Science-ction writers, according to the passage, for the most part tell us about the supposed speech and dress of aliens
(lines 711). The claim that aliens cant shoot straight isnt enough to tell us that they are aggressive, but it certainly doesnt
indicate that they are peaceful. Therefore, the claim that aliens may have used violent or oppressive means to control war
does not counter the views of science-ction writers, as they are described in the passage.
C: Yes. The researcher quoted in the second paragraph argues that any civilization that makes it into space must no
longer be warlike and aggressive; otherwise, they would have destroyed themselves (lines 1217). The statement
that alien civilizations might avoid self-destruction by means abhorrent (lines 2224) is intended to counter this
view by suggesting that a civilization might control war through aggressive or oppressive means. Therefore, as
opposed to the researchers claim, aliens who achieve space ight might still pose a threat to us.
D: This choice takes words out of context of the passage. Military tacticians are mentioned in line 52, but not as a contrast to
the idea that war can be controlled through oppressive means. If anything, given that the author states that they nd pacist
ideas to be curious or strange, we would expect that they might agree that other civilizations may display oppressive and
aggressive qualities.
97. B
Note: This question is essentially the reverse of question 94.
A: The idea that extraterrestrials who contacted Earth may be the ttest of their species is not presented as a comforting idea.
The author suggests that natural selection may favor those who are aggressive, territorial, and quick to reach for the
sword (lines 2631).
B: Yes. This is essentially stated in the passage. The author writes that in some ways the thought that natural selection
might function on other worlds as it has on ours is comforting, for [it] would imply that human nature was
something deeper even than we know. Aliensmight exhibit may recognizable traits (lines 5458).
C: According to science-ction writers (in the authors rather sarcastic description), aliens speak like Midwesterners, dress
exotically, and have bad aim (lines 711). Nothing in this description is particularly disturbing; therefore, the comforting
aspect of natural selection is not intended to counter the views promoted by science-ction writers.
D: While the discovery of an alien civilization that used nuclear knowledge to achieve space ight without destroying itself in
a nuclear Armageddon might be cause for celebration (lines 1821), the implication is that this is because their example
would suggest that we Earthlings may also be able to avoid blowing ourselves up. The author does not indicate that the
aliens would explain to us how they achieved this feat. Finally, there is no connection in the passage between avoiding
nuclear destruction and the comforting aspects of natural selection; they may even have avoided nuclear war by means
abhorrent (lines 2225).
98. A
A: Yes. The possibility that they would be warlike is described in paragraphs 3 and 4. The possibility that they might
be peaceful is described in lines 1217, and in paragraphs 5 and 6. The author does not take sides. Therefore, we
could expect them to be either one or the other.
B: The author suggests that they could also be peaceful (lines 1217 and 4261).
C: The author suggests that they could also be warlike (lines 2141).
D: If you arent sure what ethologically means (ethologically in this context would mean developmentally), focus on
the word backward. The passage never suggests that aliens are likely to be in any way backward. The passage suggests
instead that they may either be similar to humans (lines 5461), or even ahead of us, if they have in fact acquire[d] the
wisdom to control war (lines 1217).
19 Verbal Reasoning Solutions
99. C
A: James Trel argues that advanced extraterrestrials may be predators who are aggressive, territorial, and quick to the
sword (lines 2631). This is the opposite of pacistic.
B: The author describes both the view that aliens may be peaceful (lines 1217 and 4261) and the view that they could be
aggressive (lines 2141). Given that the author does not take sides, we cannot infer that the author would favor a pacistic
characterization.
C: Yes. The researchers mentioned in the second paragraph have as an article of faith that extraterrestrials will not
be hostile because they will have acquired the wisdom to control war (lines 1217). The scenario in the question
stem is entirely consistent with this view.
D: There is nothing in the scenario cited in the question stem about the aliens manner of speech, their mode of dress, or their
marksmanship. Therefore, it would neither support nor undermine the characterization favored by science ction writers
(lines 711).
Passage IV (Questions 100104)
100. A
A: Yes. The author states that under some conditions the size of the algae bloom can increase rapidly and spread
over large areas with catastrophic results (lines 2832). Out of the four choices, this is the only one that is both
supported by the passage and directly relevant to the issue of control of red tides.
B: This choice contradicts the passage. The author states that the global expansion in aquaculture means that more areas are
monitored closely (lines 5556) and that the maturation of this eld of science [is] now proting from more investigators,
better analytical techniques and chemical instrumentation, and more efcient communication among workers (lines 60
63).
C: This choice does not focus on the same issue as the question stem. The effects of red tides on sh do not indicate why the
tides themselves are difcult to control. Furthermore, the passage does not indicate that toxins are more likely to reach
dangerous levels in small sh than in medium or large sh, or that small sh are in any relevant way different than larger
sh.
D: The passage suggests that the opposite is true. See the explanation for Choice B, and lines 5556 and 6063. Furthermore,
the author states that Long-term studiesdo show that red tidesare increasing as coastal pollution worsens, in part
due to industrial, agricultural, and domestic waste (lines 6468). The existence of these long-term studies is further
indication that human pollution is in fact being monitored.
101. D
A: Given that algae blooms can spread rapidly over large areas (lines 2632), this proposal would be unlikely to signicantly
reduce the problem. It does not address the cause of red tides, and would at most offer limited protection to some
commercially farmed sh (sheries products [line 57]).
B: Aside from the fact that there is no suggestion in the passage or question stem that we are able to control the movement of
wild marine life, this would at most be a limited solution. As in Choice A, it would at most offer limited protection to some
sh and marine mammals, without addressing the cause of the problem.
C: Dumping toxic substances (herbicides) into the ocean in order to kill other toxic substances (dangerous algae) would be
a radical solution with possible unintended negative consequences. Also keep in mind that only some algae are toxic;
herbicides would kill the harmless species as well. While this choice, unlike choices A and B, would in fact address the
cause of the problem, it does so in an extreme and indiscriminate fashion. There is no evidence in the passage or question
stem which suggests that environmentalists would advise such extreme measures.
D: Yes. The passage states that when wastes, which are frequently rich in plant nutrients (lines 6768), are released
into coastal waters, we see a general increase in algal growth, including toxic algae (lines 6772). If plant nutrients
were removed from wastewater, it would help address the cause of red tides without killing off all algae or introducing
new toxins into the oceans.
MCAT Practice Test 8 Solutions 20
102. B
A: According to the passage, commercial sh are sensitive to [algal] toxins and, unlike shellsh, die before toxins reach
dangerous levels in their esh (lines 3638). Therefore people who, unlike sh, birds, and whales, rarely eat the whole
sh including the organs, are not at great risk of poisoning by red tides. Even if red tides increase, then, it does not appear
that humans would be at signicantly greater risk.
B: Yes. The passage states that Algal toxinscause mortalities as they move through the marine food web (lines
3334) because toxin accumulates in the liver and other organs of certain sh, and so animals such as other sh,
marine mammals, and birds that consume whole share at risk (lines 3841). Therefore, sh may be killed
through a direct encounter with a red tide (the author implies in lines 3538 that this is true of herring, cod,
salmon, and other commercial sh), or by eating other sh who have been exposed. Thus, if red tides increase as
the author expects (lines 6672), we would expect to see more large-scale sh die-offs.
C: The author states that red tidesare increasing as coastal pollution worsens (lines 6465) and that pollution through
nutrient-rich wastewater causes a general increase in algal growth (lines 6870). Therefore, we would expect both toxic
and harmless species of algae to increase (lines 7071).
D: The author suggests that shellsh are less sensitive than are sh to algal toxins (lines 3438). Therefore, we are more likely
to see die-offs of sh (Choice B) than of shellsh. Furthermore, even if there were one or more large-scale die-offs, this
would not necessarily mean that the total number of creatures would decrease. This choice is too extreme, especially when
compared to Choice B.
103. C
A: The question stem describes an actual epidemic of poisoning, not just groundless fears. If people were demanding more
medical attention to assuage their fear of being poisoned, it would not explain why people were poisoned in the rst place.
This choice has a problem with cause and effect. An epidemic could cause people to demand medical attention, but a
demand for medical attention could not cause an epidemic.
B: The epidemic is described as worldwide. Even in countries with a seacoast, only a minority of people would be swimming in
coastal areas. The most reasonable explanation has to explain how people in non-coastal areas are getting sick. Furthermore,
the author states that Algal toxinscause mortalities as they move through the marine food web (lines 3334). Unless
swimmers are eating signicant amounts of algae, it isnt clear based on the passage that simple exposure would cause
poisoning of humans.
C: Yes. The author states in the passage that people are generally not at high risk of red-tide poisoning because
commercial sh are sensitive to toxic algae. Therefore, these sh tend to die before toxins accumulate in the parts
of their bodies normally consumed by people (lines 3438). If food sh became less sensitive (and so were more like
shellsh [lines 3738]), toxins would have more time to accumulate and reach dangerous levels in their esh. This
esh could then be eaten by people, passing along the toxins.
D: There is no reasonable connection, based on the passage, between lower exposure of whales and other marine mammals to
toxins on one hand, and an epidemic of poisoning among humans on the other.
104. C
Note: The credited response will be the answer choice that is most inconsistent with the authors concern about the danger of
red tides. That is, we need a choice that goes the furthest to indicate that red tides do not pose a signicant danger.
A: This choice is too limited. While it may indicate that temperature uctuations, not red tide, caused the death of fourteen
whales in 1987 (lines 14 and 1213), this is not enough to suggest that red tides are generally harmless.
B: As in Choice A, this nding would suggest only that there was an alternate cause (other than red tide) in one incidentthe
death of fourteen whales in 1987 (lines 14 and 1213). Invalidating one example does not signicantly weaken the
authors overall argument.
C: Yes. This nding goes the furthest to suggest that red tides are not generally dangerous. In particular, it undermines
the implication in lines 2632 that red tides are likely to spread quickly over a large area, with disastrous results. If
red tides are rare, and if when they occur they grow slowly and can be easily identied, then they do not appear to
pose much of a threat.
D: According to the passage, it is sh, marine mammals, and birds, not humans, who are most at risk from toxic red tides
(lines 3441). Unless the antidote described in this answer choice can be easily administered to wild animals, the existence
of an antidote would not suggest that the authors concern is exaggerated. Furthermore, to the extent that humans would be
at risk, the fact that they can be treated for poisoning does not suggest that poisoning should not be a concern.
21 Verbal Reasoning Solutions
Passage V (Questions 105114)
105. C
A: This choice is too narrow to express the primary purpose. The author does discuss two different critical accounts (lines
16). However, the author goes on to describe how these two accounts came together to create a single conclusion (line
7) or myth, and then spends the rest of the passage refuting that myth.
B: As in Choice A, this statement is too narrow. The author describes the growth of the magazine industry in paragraph 3 in
order to support the larger point that pre-Civil War America was not hostile to the novel.
C: Yes. The author describes this idea (that America was hostile to both ction in general and the novel in particular) in
the rst paragraph, lists three reasons why this idea is questionable in the second paragraph, and provides further
support for the third of these reasons in paragraphs 3 and 4. The nal paragraph sums up the main idea when it
reiterates the point that the American public was not hostile to ction in general or to the novel in particular (lines
6670).
D: As with Choice A and Choice B, this choice is too narrow to be the primary purpose. The author does describe the
argument that the Puritan tradition was one aspect (the nature of American social life is another) that fostered the romance
and discouraged the novel in pre-Civil war America (lines 210) and then goes on to refute this argument (lines 1621).
However, this is just one part of the authors overall purpose, which is to argue that pre-nineteenth century America was
hospitable to ction and to a particular form of ction, the novel.
106. A
A: Yes. The author discusses periodicals reviews of novels (paragraph 4) in order to support the claim that a great
many novels were written and published in America at this time of supposed hostility to ction, and a great many
more were being read (lines 2326). If it were easy and more protable for American publishers to acquire
European novels, and yet half of the reviews were still of American novels (lines 4546), it would provide even more
evidence that there was signicant interest at that time in American novels (lines 4853). That is, it would make the
statistics cited in the passage even more compelling.
B: The author never discusses literary nationalism. Nor does the author indicate that a desire on the part of the American
public to read American novels constitutes a form of nationalism. Even if it did (which would be reading too much into
the passage), this answer choice indicates that the author argues that literary nationalism was not viable, which would
contradict that interpretation.
C: If American publishers could make more prot by publishing European novels, and if they also made prot a top priority,
one would expect that most published novels would be European in origin. However, the passage states that about half of
the novels reviewed were American in origin. Therefore, the new information in the question indicates that prot was not
a top priority.
D: See the explanation for Choice A. This information would support the authors claim that the American public was interested
in reading American novels, by suggesting that something other than prot motive must have been driving publishers
decisions to publish a signicant number of novels written by American authors.
107. A
Note: The correct answer either will not be an assertion made in the passage, or will be an assertion made in the passage but
with no support by cited historical research.
A: Yes. This choice is correct for two reasons. First, while the author does state in lines 6466 that the small number of
American ction writers who are now called major did, evidently, have trouble supporting themselves as novelists,
the author provides no historical research to back up this claim. Note that the next word after this quoted sentence
is but, indicating that the author is moving on to a different point, rather than providing further support or
explanation. Second, the passage states that these writers had trouble supporting themselves as novelists; it does not,
however, state that they had trouble supporting themselves by writing (that is, there could be ways of making money
through writing other than by writing novels). For this reason, this answer choice is not even an assertion made by
the passage author.
B: This assertion (made in lines 2329 in the passage) is supported by the research about periodical reviews described in
paragraphs 3 and 4.
C: This assertion is made and supported in lines 4863.
D: This assertion is supported by the research described in lines 2941.
MCAT Practice Test 8 Solutions 22
108. B
A: This sentence marks the end of the passages discussion of Scottish common sense philosophy. Rather than introducing a
point that is discussed or developed further later on, it follows up on a point rst raised earlier in the passage (lines 24).
B: Yes. In lines 1719 the author claims that much of what Americans wrote and said about novels was derived from
sources written in the British Isles. The sentence cited in the question stem further supports the claim that what
Americans said about ction to some degree derived from British sources (Scotland is part of Britain). Use Process
of Elimination actively; while one could debate whether the role of lines 2021 is to offer further support for the
preceding sentence or to act as a conclusion based on the preceding sentence, there is no other answer choice that
provides a more accurate description.
C: This statement is at odds with the main idea of the passage; that is, that nineteenth-century America was not hostile to
ction, including the novel. Note that the author refers to supposed hostility to ction (line 25) and expressed hostility
to ction (line 16) [emphasis added]. The purpose of the sentence cited in the question stem is to further the argument that
America was not uniquely or unusually hostile to ction and the novel.
D: While the author may (or may not) believe that Scottish common sense philosophy was to some degree hostile to ction,
the sentence quoted in the question stem does not express that view. Rather, this sentence indicates that to the extent that
any ideas drawn from Scottish philosophy appeared in or inuenced American writings about the novel, those ideas cannot
be described as essentially American.
109. B
A: The new information in the question stem describes the idea that social conditions (breakdown of cohesiveness and increase
in individual isolation) are inconsistent with the conventional novel. There is no indication that the Puritan tradition
relates to or is dened by social conditions. Therefore, Choice B, which does refer indirectly (the nineteenth-century
milieu) to similar social conditions described in the passage, is a better t.
B: Yes. The scenario described in the question stem parallels the myth described in the rst paragraph of the passage.
The author in the passage describes (and later refutes) the story that the sparseness of American social life made
the conventional novel difcult, even impossible to write (lines 46) and that the American imaginative space,
dened by an ambience of isolation, alienation, deance, and apology lent itself to the literary form of the romance
rather than to the novel (lines 712). That is, both the question stem and the passage describe the belief that when
social conditions tend towards alienation and isolation, writers will be drawn away from the novel and towards
some other literary form (such as the romance).
C: The death-of-the-novel theory is inconsistent with the research nding on book reviews described in paragraphs 3 and 4.
Those ndings, according to the author, indicate that Americans were interested both in reading and writing novels (lines
2329, 4953, and 6870).
D: The author does not present research ndings on attitudes towards the novel in England. While the author does suggest
that there was some hostility to ction in Britain (lines 1617), this claim is not supported by research cited in the passage.
Furthermore, there is no suggestion in the passage that British hostility was caused by social conditions or, more specically,
by an ambiance of isolation (line 11). Therefore, the new information is more consistent with Choice B, which does refer
to social context (the nineteenth-century American milieu).
110. B
Note: The correct answer will be the one that is most inconsistent with the passages description in paragraphs 3 and 4.
A: This is not inconsistent with the passage. The author states that there were more than two thousand reviews of eight
hundred separate novels, about half of them American in origin (lines 4446) between 1840 and 1860. While the author
argues that people in general had positive feelings towards the American novel, some did complain about overly positive
reviews or puffery (lines 5053); it would be consistent if some also complained about the number of reviews.
B: Yes. According to the authors interpretation of the research cited in paragraph 4, American novels were received
warmly by reviewers (lines 4950); this warmness reected the attitudes of Americans more generally, since novel
reviewing was conducted in constant awareness of what people were reading, and was directed toward trying to
understand the reasons for public preferences (lines 5861). This is part of the authors overall argument that
nineteenth-century America was not hostile to ction or to the novel (lines 2329). If the information in the passage
is correct, while an individual editorialist of the time could have been hostile to American ction, we would not
expect an editorial decrying American hostility to American ction. This would be denouncing something that,
according to the passage, did not exist.
23 Verbal Reasoning Solutions
C: This would not be unexpected. The author never suggests that all of the reviews of American novels originated in America.
In fact, the author states in paragraph 2 that much of what Americans wrote and said about novels was derived from
sources written in the British Isles (lines 1719). Finally, the passage does not indicate that the British were in general
hostile to American ction.
D: This would not be unexpected. The author accepts that Scottish common sense philosophy inuenced American writing to
some degree (lines 1721). There is no reason, based on the passage, to think that an essay reecting this inuence would
not appear in an American periodical of this time period.
111. A
A: Yes. Those who set themselves up as guardians of critical integrity complained that reviewers were overly positive,
i.e., writing indiscriminate puffery (lines 5053). The author goes on to state that novel reviewing was conducted
in constant awareness of what people were reading and that reviewers had a fairly precise idea of what they were
guiding and correcting through their reviews (lines 5863). These statements are intended to refute the potential
objection that these positive reviews reected only the opinions of the reviewers themselves (lines 5357). Therefore,
the complaints of the guardians provide credible evidence that the prevailing attitude of the public towards the
American novel was quite positive.
B: The word questionable disqualies this choice. The author argues that reviewers positive views (puffery according to
the self-styled guardians) reected the views of the public as a whole. The author does not suggest, however, whether or
not the positive reviews reected the true quality of the books under consideration. For the guardians ideas to be labeled
as questionable, we would need some indication by the author that the book reviews may not in fact have been overly
positive.
C: This choice goes even further in the wrong direction than Choice B. While there is no indication that the author nds the
ideas of the self-appointed guardians to be amusing, the term wrongheaded is even more problematic. The author
never suggests that the largely positive reviews did in fact reect the true quality of the books reviewed, only that they
reected public opinion of the time. Thus there is no evidence that the author believes the guardians claim that the positive
reviews were indiscriminate puffery (overly positive) was incorrect.
D: Choices B, C, and D are all incorrect because they indicate that the author disagrees to some extent with the guardians
of critical integrity; this choice (especially given the word dangerously) is the most extreme (and so the most clearly
wrong) of the three. The author does not indicate that the novels that were positively reviewed were in fact good novels,
only that the public as a whole believed that they were (lines 4950 and 5763). Therefore, there is no indication that the
author believes that the guardians of critical integrity were incorrect in their belief that reviews were overly positive
(indiscriminate puffery [line 52]). Thus, the passage does not suggest that the author believes the guardians idea to be
misinformed.
112. D
A: This choice contradicts the passage. The author states in lines 2123 that the conventional novel at that time was only in
the process of becoming conventional.
B: The passage offers no information about how common the conventional novel was is England as compared to America.
The only direct comparison we have between the two countries is that expressed hostility to ction was no less strong in
England than in America (lines 1617), but this tells us nothing about how common the novel was.
C: This choice contradicts the passage. In the rst paragraph the author describes the argument that pre-Civil War America
was inhospitable to the novel, and that the American imaginative space lent itself more readily to a different literary
form, the romance (lines 912). Even though the author disagrees with the claim that America was hostile to the novel, the
author does appear to agree that the novel and the romance are two different literary forms. Therefore, we would expect
them to be different in style and/or subject matter.
D: Yes. The author states in the rst paragraph that some literary historians believe that the sparseness of American
social life made conventional novels difcult, even impossible, to write (lines 46). This suggests that novels are
based on observations of social life, and that when there is little to observe, it is difcult to write novels. The author
does go on to argue that novels were in fact quite common in this time period, but the passage never directly denies
a connection between social life and the novel. Use Process of Elimination; this is not a great answer, but it is the
best of the four choices.
MCAT Practice Test 8 Solutions 24
113. D
Item I: True. This piece of information would be valuable in evaluating the accuracy of the authors claim that the
generally positive reaction of reviewers to American novels represented a similar opinion on the part of the
American public (lines 4863).
Item II: False. While the author mentions that the reviews varied in length (lines 4648), the author doesnt make an argument
correlating or relating length of a review to popularity of the book. Therefore, this piece of information would not
be relevant to evaluating the authors conclusions.
Item III: True. The author refutes the claim made by literary historians that The would-be American novelist before
the Civil War was drawn, or forced, toward a literary form better suited to the American imaginative space:
the romance (lines 710). The author refutes this claim in part by citing evidence that many popular American
novels were written in this time (lines 2326 and 4263). It would be useful to know, then, the popularity of
romances among American readers; if romances were much more popular than novels, it would support the
literary historians story over that of the author. If, instead, novels were much more popular than romances,
it would strengthen the authors conclusions in the passage.
114. C
A: The author never argues that the romance was not uniquely American. The only direct mention of the romance appears in
lines 1012. While the author does go on to argue that the idea that the romance was better suited than the novel to America
in that time period is a critical myth (lines 1316), the author makes this argument by claiming that many novels were
written in America, not by claiming or implying that many romances were written in other countries.
B: There is nothing in the new information or in the passage to suggest that Wuthering Heights is not a romance. Furthermore,
there is nothing in the new information or in the passage to suggest that the literary historians mentioned in the rst
paragraph believe that romances were only written in America. Therefore, this new fact would not demonstrate that literary
scholars are mistaken even occasionally, much less often. Finally, even if we did have evidence that literary historians
are often mistaken, it would not signicantly strengthen the passage authors argument that literary historians are mistaken
about the novel in nineteenth-century America (lines 116).
C: Yes. See the explanation for Choice A. The author makes no claims about whether or not romances were written in
other countries. The only claim the author makes relative to the romance is that the romance was not better suited
than the novel to the American imaginative space between 1840 and 1860 (lines 1316).
D: Even if an atmosphere of alienation had spread to England from America, it would not undermine the passage argument.
The author states that some ideas, for example Scottish philosophy, spread from England to America (lines 1621); it
would not be inconsistent if ideas owed in the other direction as well. Furthermore, even though some literary historians
connect the romance to an ambiance ofalienation (lines 1011), one romance written in England would not constitute
evidence that an atmosphere of alienation existed in that country.
Passage VI (Questions 115122)
115. D
A: This statement is consistent with the authors claim that by the late 1960s and early 1970s, limits on the energy base [in the
United States] began to surface (lines 24). The author also mentions the decline in U.S. oil production after 1970 (lines
2426).
B: This statement is consistent with the passage. The author states that Environmentalism made its inuence felt in a large
number of ways (lines 45) and then describes a variety of environmental legislation in lines 511. The passage also
describes how environmentalism led to a switch from coal to low-sulfur oil (lines 1216).
C: Although the author states that the fact that the U.S. was outrunning its geological basewas not represented in either
the consumption patterns or in prices (lines 2224), the authors description of 1970 as a turning point when U.S. oil
production peaked and then declined (lines 2426) would be consistent with the statement that use of domestic oil has
declined in recent years. This answer choice would also be consistent with the authors claims that an increasing number
of individuals and communities in the U.S. are shifting to the soft path (lines 4041) and that low-sulfur oil is being
imported (line 16).
D: Yes. This choice is inconsistent with the passage. The author claims that A more rapid spread of [the soft path]
approach is being hindered by governmentsubsidies of the hard-path approach (lines 4143).
25 Verbal Reasoning Solutions
116. D
A: Lines 5466 describe how large corporations are buying up copper mines and solar technology companies, while at the
same time disparaging solar energy. This paragraph has a negative tone, suggesting that the oil companies and other
large corporations want to control the spread of solar technology in order to protect their own prots (lines 6063).
Therefore, the passage suggests that the author would oppose, not support, acquisition of solar technology companies by
large corporations.
B: The passage suggests that the author supports increased use of soft energy technologies which use to the greatest possible
extent nondepletable sources like sun, wind, and vegetation[and] emphasize diversication and dispersal of energy
sources so as to avoid in the future the sort of dependence we now have on fossil fuels (lines 3539). In particular, the
author mentions solar energy as a promising source of energy (lines 4852). Even though low-sulfur oil pollutes less than
coal (lines 1316), it is still a fossil fuel and part of the hard energy path.
C: See the explanation for Choice B. The author appears to support increased use of soft energy (lines 3539 and 4852).
Nuclear reactors are listed as part of the hard path (lines 3034).
D: Yes. The author criticizes the government for hindering the spread of the soft path by subsidizing the hard path
(lines 4143). The author also suggests that solar energy has the potential to supply a signicant percentage of our
energy needs (lines 4852), and yet there is inadequate access to capital for development of solar energy resources
(lines 4647). Therefore, we can infer that the author would support federal subsidies for solar energy.
117. C
A: This choice is inconsistent with the fact that Concern about air pollution led to fuel switching, especially by electric
utilities, away from domestically produced coal to low-sulfur oil (lines 1316).
B: The author never cites the environmental movement as insufciently responsive; in fact, the environmental movement
appears to have had a signicant impact on legislation (lines 411). Rather, the problem with shifting to a more
environmentally friendly soft path lies with the government, building codes, lack of capital, false beliefs about solar
energy, and large corporations (lines 4152 and 5866).
C: Yes. In the last paragraph, the author describes how oil companies are buying up copper mines, and how large
corporations began buying up solar technology companies (lines 6566) while they still disparaged solar energy
(lines 5960). By claiming that these actions were motivated by a concern that every rooftop could become its own
power plant and sensing that the cry for solar energy was a revolt against huge companies, utilities, and staggering
electric bills (lines 6063), the author suggests that large corporations acted primarily to protect and maximize
their own prots. Use Process of Elimination aggressively; this choice is a bit extreme, but it is the best supported of
the four choices.
D: The author directly states that a more rapid spread of [the soft path] approach is being hindered by government (taxpayer)
subsidies of the hard-path approach (lines 4143). Therefore, even though the government has passed some pro-
environment legislation (lines 511), there is evidence that the Congress is not fully committed to reform.
118. B
Item I: True. Solar energy is part of the soft path (lines 3536), and spread of this soft path is held back in part
by lack of capital (lines 4647) and by government subsidization of hard-path technologies (lines 4143).
Therefore, we can infer that an advocate of the soft path would support tax credits for implementing solar
technology.
Item II: True. An advocate of the soft path would certainly support legislation mandating a proportional increase in
the use of soft energy.
Item III: False. Gas is listed as part of the hard-path solution (lines 3033). We have no reason to think that soft-energy
advocates would support switching from one hard-path resource to another. Be careful not to use outside knowledge
or personal opinion. Also be careful to read the choice carefully though the last word; this choice sounds promising
until the word gas at the very end.
MCAT Practice Test 8 Solutions 26
119. B
A: This choice contradicts the passage. The author states, In 1984, for example, about 18% of all primary energy used in the
world and 8.5% of that used in the U.S. came from renewable solar energy resources (lines 5052).
B: Yes. The author states that Concern about air pollution led to fuel switchingaway from domestically produced
coal to low-sulfur oil and that this oil had to be imported (lines 1316). While this is not a perfect answer (it
assumes that all or most imported oil is low-sulfur and that non-domestic coal pollutes as much as domestic coal), it
is the best of the four choices.
C: This choice is too extreme (and is more extreme than Choice B). The passage states that oil companies now control almost
60% of domestic copper production in the U.S. (lines 5658). However, we know nothing about their control on a global
level.
D: The author argues that the usual proposed hard-path solution is the rapid expansion of three sectors (lines 3031),
and gas is listed along with oil as the second of those three (lines 3233). Even though more and more individuals
and communities in the U.S. are shifting to the soft path (lines 4041), many barriers are also described (lines 4150).
Therefore, we cannot infer that consumption of natural gas has declined.
120. D
Note: If the soft-energy path is fully implemented, that means that hard-path energy is no longer used. Therefore, the correct
answer will describe something connected to outdated hard-path energy use.
A: The author lists wind as a soft-path resource (lines 3536).
B: Conservation is consistent with the soft-path (lines 4145).
C: Corn, an example of a nondepletable resource like sun, wind, and vegetation (lines 3537), would be soft-path.
D: Yes. Antipollution devices on smokestacks would be much less likely to be necessary when dealing with soft energy,
which the passage implies is less polluting than hard energy sources like coal and oil.
121. A
A: Yes. The new information in the question relates to the theme in the last paragraph in the passage. In that paragraph,
the author describes how oil and other large companies have bought up copper mines and solar technology companies
in order to control the implementation of solar technology. If an inventor perfected a solar battery, then, it would
be likely that utility companies (mentioned or referred to in the passage in lines 5859 and 62) would seek to control
that technology.
B: The author claims that government subsidization of the hard-path is hindering spread of the soft-path, including solar
energy (lines 4143). Therefore, it is unlikely that the government would suddenly switch sides to subsidize solar energy.
C: The passage suggests just the opposite: energy and high technology companies disparaged solar energy and spent a
share of their public relations budget playing down the solar messiahs (lines 5865).
D: While this choice does not contradict the passage, at the same time there is no support for it in the passage text. Compare
this choice to Choice A. We have an analogous case in the passage (paragraph 5) that makes Choice A likely to occur. On
the other hand, the passage never describes lobbying efforts on the part of hard-path (including nuclear energy) industries.
We dont know, in particular, that hard-path industries lobbied the government to obtain subsidies or to maintain outdated
building codes (lines 4146).
122. A
A: Yes. While the passage does not discuss a relationship between price and consumption, this is the only choice that
is consistent with the new information in the question. That is, it is the only choice that describes a disadvantage
attached to a hard-path resource. It makes sense that if the price of oil, a hard-path resource, rose precipitously,
people would be more likely to seek out soft-energy alternatives.
B: If nuclear plants (hard-path [line 34]) were made safer, this if anything would encourage the continued or increased use of
that hard-path resource.
C: If we could use coal without polluting, this would encourage, not discourage, the continued or even increased use of that
hard-path resource.
D: The ability to fulll our needs without having to import oil would not encourage us to seek elsewhere, in the soft-path, for
energy sources.
27 Verbal Reasoning Solutions
Passage VII (Questions 123127)
123. A
A: Yes. The passage describes sports as the most powerful manifestation of the godward signs in contemporary life
(lines 5455), and as manifestations of concern, of will and intellect and passion (lines 6062). The passage as a
whole, in its description of sport as a civil religion (lines 12) leads up to this claim. Therefore, it is reasonable to
conclude based on the passage that one should recognize the profound importance of sports. While this would be
an even better answer choice if it mentioned the religious aspect of sport, even as written it is the best of the four
choices.
B: While the passage does discuss how the ancient Olympics did honor the gods (lines 24), the passage as a whole does not
discuss the religious history of sports. Rather, it discusses how sport has served a religious function (line 49) that may
have little or nothing to do with God or gods (lines 5559), or with institutionalized religion.
C: While patriotic impulses are mentioned in the rst paragraph, the passage as a whole describes the importance of the
religious function of sports. This choice is too narrow to express the primary message of the passage.
D: Nowhere in the passage does the author discuss the importance of distinguishing between these two aspects. Rather,
ceremony (lines 4249) and excitement (lines 4546) are both mentioned or suggested, along with many other qualities, as
aspects that are interwoven within the natural religion (lines 3334) of sports.
124. B
A: Be careful not to use outside knowledge or personal opinion. The author explicitly states that by godward he does
not mean belief in any concept of God (lines 5457). Rather, godward suggests a drive towards manifestations of
concern, of will and intellect and passion (lines 5962). In the previous paragraph, the author refers to a pagan sense of
godliness (line 53) as part of the religious function of sports (line 49). Therefore, the word godward, as used in the
passage, does not refer to a divine presence.
B: Yes. Throughout the passage the author makes it clear that he is not using the word religious in the usual
way; that is, to refer to a particular faith or form of worship. Rather, sports constitute a natural religion that
can manifest itself in a wide variety of ways, from the asceticism and dedication of preparation to a sense of
participation in the rhythms and tides of nature itself (lines 3341). Even agnostic persons (line 48) can partake
of this experience. The author takes great pains to show how all kinds of people can experience this religious impulse
in a great diversity of forms. The authors use of the word godward continues this theme, as the author writes, I
dont mean that participation in sports, as athlete or fan, makes one a believer in God, under whatever concept,
image, experience, or drive to which one attaches the name (lines 5559).
C: This choice is inconsistent with the passage. The author explicitly states that godward does not refer to the worship of
God, as would be the case in conventional religion. Popular spiritual movements are never discussed by the author.
D: The author argues throughout the passage that sport arises from and itself constitutes a religious godward impulse that
is distinct from religious movements and institutions. Therefore, attending sporting events would not lead to a revival of
religion; attendance at sporting events would itself constitute religious practice.
125. C
A: While the author states that the ancient Olympics and sports ever since have incorporated both political and religious
aspects (lines 27), he does not suggest that the athletes themselves are either religiously devout or patriotic. In fact, later in
the passage the author states that The athlete may of course be pagan (lines 3233). Finally, the author does not suggest
that he would respect athletes for being either devout or patriotic.
B: The author never mentions skill of the athletes in this context.
C: Yes. The rst paragraph describes how the Olympics, from the ancient to the modern games, have been religious
and political as well as athletic events (lines 25 and 1316). The rest of the passage argues that a religious impulse
(not necessarily dened as a faith in a divine being) is a fundamental and essential aspect of sports. Therefore, by
stating that the ancient games were held in honor of the gods andin honor of the stateand that has been the
classical position of sports ever since (lines 35), the author is describing the historical function of athletics.
D: The word domination is not consistent with the authors description. Sports has always (since the ancient Olympics)
incorporated both religion and politics, but the author does not indicate that athletics has been overshadowed by religion
or politics.
MCAT Practice Test 8 Solutions 28
126. C
Note: The credited response will be inconsistent with the authors description of the religious impulse (as he denes religious)
inherent in sports; it is this impulse which denes the importance of sports for the author.
A: This choice is not inconsistent with the passage. The author states, Most men and women dont separate the sections of
their mind. They honor their country, go to [a place of worship], and also enjoy sports (lines 1719). A nding that people
prefer religious ceremonies to sporting events would not weaken the authors argument that sports has its own religious
aspects.
B: See the explanation for Choice A. The authors argument does not depend on the capacity of sports fans to recognize the
religious function of sports. In fact, the author suggests that most people do not separate the sections of their mind (lines
1718); that is, they dont necessarily see the role played by religion and politics in sporting events.
C: Yes. If only conventionally religious people experienced the spiritual dimension of sports, this would suggest that the
natural impulse that is radically religious (line 30) and the godward drive of sports (lines 5460) do not include
and apply to all participants, including those who do not believe in a God (lines 3233 and 5559). In this case,
the religious value of sports would be much more limited than the author asserts.
D: This claim has no direct relevance to the authors assertions about the value of sports; those assertions do not rest on the
claim or assumptions that fans have stronger feelings (religious or otherwise) than the players. The author discusses the
feelings of both (see lines 3041 and 5559), without prioritizing one over the other.
127. A
A: Yes. Immediately before that sentence, the author states that sports are manifestations of concern, of will and
intellect and passion (lines 6062). If losing is a symbolic form of death, this would highlight the level of concern
and passion, or emotional investment, entailed in sports.
B: This choice is out of scope. The passage does not suggest that losers should be given equal honors.
C: As in choice B, this answer choice is out of scope of the passage. Painful and unacceptable are not necessarily the same
thing. In fact, the author suggests a certain nobility to loss in this context (lines 6063).
D: The word exaggerated suggests a critical attitude that is not expressed by the author. The passage does not indicate that
experiencing a loss as a kind of death is inappropriate. In fact, the author indicates that the intense religious experience
provided by sports serves an important function (lines 4953).
Passage VIII (Questions 128132)
128. C
A: Only type-type identity theory is described as reducing mental states to physiological states (every type of mental
[experience], such as pain, is identical to a corresponding type of physical [event], such as the ring of C-bers (lines 33
36). While behaviorism is described as simple (lines 1213), it is described as reducing mental phenomena to behavioral
dispositions or tendencies (lines 25), not to physiological states.
B: First, type-type identity theory does provide a reason for the existence of pain, that is, the ring of C-bers (lines 3336).
Second, behaviorism provides a reason as well. A mental state such as pain in the big toe consists of a set of dispositions to
favor the big toe in certain ways (lines 56) and the belief that it is going to rain consists of dispositions to act as if it was
going to rain (lines 1011); therefore, according to behaviorism, mental states or beliefs are constituted by dispositions to
act in certain ways.
C: Yes. The author argues that behaviorism should be rejected in part because it ignores how beliefs interact with
each other and with various desires and other mental states to create particular behavioral dispositions (lines
5964). The author suggests that type-type identity theory has the same failing. In this theory, every type of mental
[experience]is identical to a corresponding type of physical [event] (lines 3335). That is, one mental experience
is caused by one physiological event; there is no consideration of how beliefs and feelings might interact with each
other to lead to particular outcomes. This is one reason why type-type identity theory implies that the same (internal)
physical event would cause the same mental event in any kind of creature (lines 3639); the different contexts
in which animals and people exist would be irrelevant. Note that type-type identity theory is distinguished from
functionalism (beginning in line 39). Paragraph 5, which does describe taking context into account, is discussing
functionalism, not type-type identity theory.
D: This is true, according to the author, of behaviorism (lines 1724). However, type-type identity theory holds that mental
events are caused or determined by internal physiological events (for example, the mental event of pain is caused by the
physical event of the ring of C-bers inside the body [lines 3336]).
29 Verbal Reasoning Solutions
129. B
A: According to the passage, behaviorists believe that pain (or, by implication, relief of pain) consists of certain behavioral
dispositions and actions (lines 211). In the new information in the question stem, pain relief is caused by the release of a
substance within the body; nothing in this new information relates to the behaviorist explanation of mental events.
B: Yes. According to type-type identity theory, each mental event, such as pain, corresponds to a particular internal
physical event (lines 3336). The author also states that this implies that the same physical event would cause the
same mental event, regardless of whether the physical event was occurring within an animal or a human being
(lines 3639). Therefore, someone who argues that because a substance that relieves pain in humans was found in
earthworms, earthworms must be able to feel pain relief (and so, pain) would likely be a type-type identity theorist;
one type of physical event corresponds to one type of mental event.
C: Functionalists, according to the passage, look at a whole system of interdependent functions (lines 4951). That is, to
understand beliefs and other mental states (including pain), you have to understand [the way] beliefs function in relation to
desires, intentions, perception, emotion, and inference (lines 5558). Unless all earthworms have the exact same desires,
intentions, etc. as all humans, a functionalist would argue that nding a physical substance that relieves pain in humans
tells us little about whether or not that substance corresponds to the same mental state (pain relief) in earthworms.
D: See the explanation for Choice B. While the author (who appears to support functionalism) would likely critique the
conclusion described in the question stem (lines 3639), the author does not go so far as to suggest that type-type identity
theorists do not deserve to call themselves scientists. The author never argues that this approach is non-scientic.
130. A
A: Yes. The author begins a critique of behaviorism in the second paragraph. The passage describes the view of
many critics who believe that behaviorism takes too external view of the mental and that mental states and
processes are not behavioral dispositions; they are the internal states and processes that are responsible for relevant
behavioral dispositions (lines 1722). It becomes clear in the following paragraph that the author agrees with these
critics when the passage states that Now it seems clear that states and processes [of] the brain are responsible for
behavioral dispositions (lines 2324). The author continues this critique in the nal paragraph, when he writes that
crude behaviorism must fail because it does not consider how relevant behavioral dispositions are determined
by various beliefs taken together plus various desires and other mental states (lines 5964). That is, behaviorism
fails to consider this internal context.
B: The author criticizes type-type identity theory by suggesting that it presents an overly strong thesis because of its implication
that the physical basis for any type of mental [experience] must be the same in different creatures (lines 3639). The
author follows this with a description of a [weak form of] functionalism; it becomes clear in the nal two paragraphs that
the author tends to agree with functionalism. In particular, the author uses notions proposed by functionalism to criticize
behaviorism (lines 6469).
C: Predictable actions are never discussed in the passage.
D: According to the author, crude behaviorism accepts that feelings and beliefs do exist, and claims that all mental states
including beliefs and feelings are behavioral dispositions or tendencies (lines 25). For example, a behaviorist would
claim that the belief that it is going to rain consists of dispositions to act as if it was going to rain (lines 1011).
MCAT Practice Test 8 Solutions 30
131. A
Note: The credited response will be the choice that is most inconsistent with the passage.
A: Yes. This choice describes a conclusion that would be drawn by a behaviorist, not by the author. Behaviorism
identies beliefs with actions. According to this theory, if the climber accepts another rope (action), that would
constitute evidence for a particular belief (weakness of the original). The author criticizes behaviorism in part by
arguing that A particular belief taken by itself cannot be identied with particular behavioral dispositions because
the relevant behavioral dispositions are determined by various beliefs taken together plus various desires and other
mental states (lines 6064). The example of the umbrella (lines 6469) continues this argument that a single action
may be determined by a whole set of interrelated beliefs and desires. Therefore, the author would be unlikely to
agree with the conclusion described in this choice.
B: The author would likely agree that a particular belief (weakness of the original rope) may lead to a variety of different
results, depending on how that belief interacts with other beliefs and desires (lines 6069).
C: As in Choice B, this conclusion is consistent with the authors argument that actions or behavioral dispositions (taking the
new rope) are determined by the interaction of a variety of different beliefs and desires (lines 6069). Therefore, the author
would agree that a belief in the weakness of the original rope would not by itself determine the climbers decision.
D: The author would agree with this conclusion; the passage states that it seems clear that states and processes [of] the brain
are responsible for behavioral dispositions (lines 2324).
132. A
A: Yes. If there is a need for a belief, that belief would perform a certain function, and Functionalism holds that
mental events of a certain sort are to be identied as those underlying events, whatever they are, that function in the
relevant way (lines 4244). While the author uses examples of beliefs we would tend to see as explicit to describe
functionalism in the passage (e.g., yourbelief that umbrellas are good for keeping rain off [lines 6768]), nothing
in the passage suggests that functionalism is limited to explicit beliefs or information. Note: In this context, implicit
beliefs would refer to beliefs that are only articulated when a relevant occasion arises. People dont generally walk
around with the well-formed belief that giraffes dont wear hats in the wild because most people would never have
considered the issue. However, most would agree that wild giraffes dont wear hats, based on general knowledge, if
a response to the question of whether or not giraffes wear hats were required.
B: This response is consistent with type-type identity theory (lines 3339), not functionalism. According to functionalism,
beliefs are neither dispositions to behave (behaviorism) nor underlying physical events (type-type), but instead are mental
events that in conjunction with other mental events determine or cause behavior (lines 4248 and 6064).
C: First, the author does not distinguish between implicit and explicit. Second, the author argues that beliefs are quite important
within functionalism (lines 6264), along with desires, intentions, perception, and feelings (lines 5558), that is, along with
other mental states.
D: As described in the question stem, the implicit belief that wild giraffes dont wear hats does have a function; it allows
someone to answer the question of whether or not giraffes wear hats in the wild. The author does not suggest in the passage
that functionalism distinguishes between implicit and explicit beliefs, and beliefs in general play an important role in the
functionalist view (lines 6269).
Passage IX (Questions 133137)
133. C
A: This choice is inconsistent with the passage. Underusers are people who rarely use their learned competence or perhaps
those whose learned competence is low (lines 4446). It would make little sense to say that learned rules are most useful
when learned rules are not being used.
B: The passage directly states that learned rules or such (cognitively based) knowledge is usually not very helpful in normal
communicative situations, particularly in the beginning stages (lines 3234).
C: Yes. The author writes, Most speakers meet conditions for monitoring regularly and systematically only on cognitive
grammar tests (lines 2527). Monitoring is dened as editing oneself to make minor changes or corrections
based on learned rules (lines 1519).
D: This choice contradicts the passage. The author states that learned rules are usually not very helpful in normal communicative
situations (lines 3234), like conversations.
31 Verbal Reasoning Solutions
134. B
A: Superusers are those who are consciously able to apply learned rules quickly and efciently so that a listener would
not notice the monitoring at all (lines 5658). Someone who frequently corrected him or herself while speaking would not
qualify as a superuser; the monitoring would be noticed.
B: Yes. Overusers spend so much time and effort on correctness that it often seriously interferes with communication
(lines 4850). Someone who frequently corrected his or her own grammar while speaking would qualify as an
overuser.
C: Optimal users are those who are able to monitor their speech and improve their level of grammatical accuracy, but not to
such an extent that it interferes with smooth communication (lines 5154). It is likely that frequent self-correction would
in fact interfere with smooth communication.
D: Underusers are those who rarely use their learned competence or perhaps those whose learned competence is low
(lines 4446). One would expect an underuser to rarely if ever correct his or her own grammar while speaking.
135. A
Note: The credited response will be inconsistent with some claim made by the monitor theory, as that theory is described in the
passage.
A: Yes. The monitor theory as formulated by Krashen and others indicates that students who wish to communicate
must acquire this ability in much the same way that speakers, adults or children, acquire it in natural situations
(lines 1215). Furthermore, learned, rather than acquired rules are of limited use to the student; for some, they
serve as a monitori.e., primarily an editor to make minor changes or corrections in utterances which for the
most part are initiated by acquired knowledge (lines 1520). The passage also states that according to the monitor
theory, Most speakers meet conditions for monitoring regularly and systematically only on cognitive grammar
tests (lines 2527) and so that oral prociency in communication is not necessarily related to the ability to achieve
high scores on standard grammar tests (lines 3537). If the scores of beginning students on grammar tests did in
fact predict their oral prociency, this would cast doubt upon the monitor theorys claim that learned rules are
only useful for grammar tests. It would also indicate that grammar tests may be a good way of evaluating students
progress (lines 3741). Finally, it would also suggest that learned rules are in fact useful in the beginning stages of
learning a second language, as opposed to the monitor theorys claim to the contrary (lines 3234).
B: See the explanation for Choice A. This choice would strengthen, not weaken the claim that learned rules are usually only
useful on grammar tests (lines 2527).
C: Superusers are unusual in that they can seamlessly monitor their own speech when necessary (lines 5558). Many
language instructors fall into this category (lines 5860). One would expect these instructors scores on grammar tests to
match their oral prociency. The author states that many, perhaps most, students are not capable of performing with the
mental gymnastics of their supermonitor instructors (lines 6466).
D: While many language instructors fall into the superuser category, some or many others would not. For those who are
not superusers, we would expect that their scores would be poor predictors of oral prociency (lines 3437). This nding
is not inconsistent with the monitor theory.
136. A
A: Yes. In the beginning of the passage, the author describes acquisition as the unconscious formulation of
grammatical principles (lines 34). The author goes on to suggest that students who wish to communicate must
acquire this ability in much the same way that speakers, adults or children, acquire it in natural situations (lines
1215). Therefore, underusers (those who rarely use learned grammatical rules [lines 4446]) may still learn to
speak well through acquisition, that is, to learn like children through natural experience.
B: Underusers rarely use learned rules of grammar (lines 4446). The fact that learned rules are not particularly useful does
not explain how underusers can in fact become competent speakers through some other route (acquisition). That is, the fact
that underusers do not use a relatively useless means to an end does not explain how they eventually succeed in achieving
that end.
C: People who can seamlessly apply rules to monitor and correct their own speech would be superusers (lines 5658). This
information does not help us to understand how underusers (on the opposite end of the spectrum) might achieve high
levels of communicative accuracy.
D: Modications to utterances, as described in the passage (lines 1720) entail applying learned rules to acquired knowledge.
Given that underusers rarely use learned rules (lines 4446), this information does not explain how underusers may still be
able to learn to communicate well.
MCAT Practice Test 8 Solutions 32
137. C
A: The passage suggests that conscious learning is not an effective way of learning a second language (lines 717 and
2834). The author specically states that learning rules through drill (line 30) is usually not very helpful in normal
communicative situations, particularly in the beginning stages (lines 3234). Memorization of vocabulary through drilling
would therefore be unlikely to improve uency.
B: The author suggests that the best way to become uent, including the acquisition of grammatical rules, is to acquire
[this ability] in natural situations (lines 1115). At the end of the passage, the author also states that most adults can
acquire grammatical rules if they interact in communicative situations with native speakers of the target language (lines
6872). There is no natural situation or interaction entailed in using an audiotape while sleeping. This choice may be
tempting because listening while asleep may appear to engage the unconscious rather than the conscious mind, and the
author indicates that the unconscious acquisition of grammar is much more effective than conscious learning (lines 310).
However, the author never discusses this particular form of unconscious learning, and it is still missing the crucial element
of using a language in a natural and interactive context.
C: Yes. The author states that students who wish to communicate must acquire this ability in much the same way that
speakers, adults and children, acquire it in natural situations (lines 1215). At the end of the passage the author
comes back to the same theme, stating that most adults acquire rules readily, although usually imperfectly, given
the chance to interact in communicative situations with native speakers of the target language (lines 6872). While
they may not achieve perfection, using language in an interactive, natural situation (in this answer choice, by asking
questions) is, according to the author, the best way to attain uency.
D: The author argues that memorization (conscious learning) is not an effective way of learning a second language. Acquisition,
which is the unconscious formulation of grammatical principles is the more effective route (lines 110). This acquisition
is most likely to occur in a natural situation (lines 1215) when students can interact with native speakers (lines 7072);
this would not be the case when a student is performing mental exercises to improve memory.
33 Biological Sciences Solutions
BIOLOGICAL SCIENCES SOLUTIONS
Passage I (Questions 138143)
138. D. The passage states that rickets is a disease causing inadequate mineralization of bone, and is due to insufcient
vitamin D activity. It also states that parathyroid hormone leads to bone breakdown. Thus, Statement I is false: a deciency
of parathyroid hormone would not lead to inadequate mineralization, it would lead to less bone breakdown. This eliminates
answer choices A, B, and C, leaving D as the correct answer. Statements II and III are true; both an inability to convert vitamin
D to its active form and the inability of active vitamin D to affect its target tissue would lead to inadequate mineralization. Note
that this is a common theme on the MCAT: a faulty messenger or a faulty receptor both lead to the same physical symptoms.
139. B. The passage states that vitamin D stimulates the absorption of calcium from the small intestine (choice B is correct
and choice D is wrong). Since calcium supplements are typically swallowed, including vitamin D in the tablet would enhance
the uptake of that supplement. Choice A is true but irrelevant; it supports the need for vitamin D supplementation, but does
not explain why vitamin D should be included in a calcium supplement. Activated vitamin D enhances parathyroid hormone
activity, not calcitonin activity (choice C is wrong).
140. B. Statement I is true; enhancing osteoclast activity will increase bone breakdown thereby releasing more calcium into
the blood. Statement II is also true; parathyroid hormone is the trigger for enhanced osteoclast activity, and it is released when
blood calcium levels are low. Statement III is false; although vitamin C is required for adequate bone formation, it has no role
in regulating blood calcium levels.
141. C. Calcitonins job is to decrease bone resorption. If the level of calcium in the plasma is high, there is no need to resorb
bone; thus, calcitonin is released. A dietary deciency of either calcium or vitamin D would lead to low plasma calcium levels,
and under those conditions bone resorption would need to remain high (choices A and B are wrong). Because parathyroid
hormone and calcitonin have opposite effects, calcitonin cannot compensate for low parathyroid hormone levels (choice D is
wrong).
142. B. A child with rickets has a vitamin D deciency. The passage states that vitamin D is nonpolar, so dietary deciencies in
fat-soluble vitamins would most likely result in vitamin D deciency (choices C and D are wrong). Furthermore, a child living
in a northern climate is more likely to develop rickets than a child living in a tropical climate, because the child in the tropical
climate would have enough sunlight available to stimulate vitamin D production by the skin. This could compensate for a lack
of vitamin D in the diet (choice B is correct, and choice A is wrong).
143. A. Removal of the parathyroid glands would result in a lack of parathyroid hormone, which is needed to increase plasma
calcium levels when they are low. Calcium is required for the proper function of neurons (neurotransmitter release) and muscle
(excitationcontraction coupling). If the parathyroid glands were removed, the body would not be able to x low plasma calcium
levels, and neurons and muscles would cease to function properly. Calcitonin has opposite effects in the body from parathyroid
hormone and cannot compensate for its loss (choice B is wrong). Drastic changes in the mineral-to-matrix ratio describes
rickets, which is not caused by a lack of parathyroid hormone (choice C is wrong). There would not be an accumulation of
calcium in the plasma; if anything, there would be a deciency (choice D is wrong).
Passage II (Questions 144150)
144. B. The best evidence for the necessity of something in the body is obtained by taking away that thing and seeing if
the effects are still present. Thus, repeating the experiment in an animal that lacks neutrophils and observing the amount of
resulting tissue damage would best clarify the role that neutrophils play in this process. Repeating the experiment with B-cell
depleted animals would give evidence as to the role of B-cells but would not give information regarding the role of neutrophils
(choice A is wrong). Repeating the experiment with different antibodies is a variation of the original experiment and will give
more evidence, but the evidence would not be as strong as in the experiment removing the neutrophils altogether (choices C
and D are wrong).
MCAT Practice Test 8 Solutions 34
145. A. The passage states that the adherence of neutrophils was facilitated by a receptor on the neutrophil, and that adherence
resulted in tissue damage. Antibodies against the subunit prevented tissue damage, implying that it prevented adhesion, while
antibodies against the subunit had no effect (neutrophils must still have been able to adhere). There is no reason to assume
that proteinases are transferred to the neutrophils; if anything, proteinases are being transferred from the neutrophils to the
surrounding tissue, resulting in damage (choice B is wrong). Since the bonding between the two subunits was described as
noncovalent, it is safe to assume it involves hydrogen bonding; however, this would not be the sole function of the subunit
(choice C is wrong). Since this is a normal receptor on the neutrophil surface, it should not result in antibody formation in a
normal animal (antibody formation should occur only in the presence of foreign or abnormal proteins, choice D is wrong).
146. D. The graph shows that a reduction in tissue injury occurred only when the antibody was received before reperfusion
(note that before ischemia implies before reperfusion). Animals that received the antibody during reperfusion were not
helped; they still had a high percentage of tissue injury (choice D is correct, and choices A and C are wrong). Applying the
antibody after reperfusion was not tested in this experiment (choice B is wrong).
147. B. Antibody B is directed against the receptor that leads to adhesion and release of toxic products. The antibody would
bind to the receptor, preventing adhesion and the subsequent effects. The passage offers no information about the afnity of the
antibody, and further, the afnity of an antibody has nothing to do with whether or not it would be rejected by the patient (choice
A is wrong). It is true that antibodies are very specic, so an antibody directed against the subunit will not bind anything other
than the subunit. However, this does not explain why it would be better than free-radical or protease inhibitors (choice C is
wrong). The passage offers no information about the half-life of antibody B (choice D is wrong).
148. B. The human body will generate antibodies against any foreign protein, including mouse antibodies. However, initial
responses to foreign proteins are generally weak compared to subsequent responses to the same foreign protein; thus, with
repeated usage the human response to the mouse antibody would get more and more severe, even if the initial response was
weak (choice B is correct, and choice A is wrong). Antibodies will recognize whatever they are generated against, regardless
of the organism they were generated in (choice C is wrong). The passage demonstrates the use of the antibody in vivo (choice
D is wrong).
149. B. Pus results when neutrophils destroy bacteria by phagocytosis. It is a mix of dead cells and cellular debris. Thus, the
presence of pus indicates the presence of neutrophils doing their job, and in order for the neutrophils to be present, they must
have functional adhesion receptors (choice B is correct, and choice A is wrong). The amount of circulating red blood cells has
nothing to do with neutrophil adhesion receptors (choice C is wrong), and if the neutrophil adhesion receptors were defective,
more white blood cells would be circulating because less would be adhering (choice D is wrong).
150. C. If the neutrophils can migrate through vascular endothelium, they must still be able to bind to the endothelium. If they
are binding, they are probably still able to release their toxic products (choice A is wrong). Since we know that the subunits
are necessary for binding etc., we can assume the subunits are functional, but this does not tell us that only the subunits are
functional (choice B is wrong). If the cells are migrating, they must have a functional cell membrane (choice D is wrong).
Passage III (Questions 151155)
151. C. Kinetic control means the product that forms faster is the major product; the rate at which a product is formed is
dependent on the activation energy for the reaction. Therefore, the product with the lower activation energy is formed faster.
Choice D states exactly the opposite, and can therefore be eliminated. Choice B is a false statement since rate of formation is
dependent on the activation energy required to form a specic transition state. Choice A can be eliminated since melting point
has no connection to rate of formation of a product.
152. C. Thermodynamic control means that the more stable product will be formed. Choice D can be eliminated; if the
molecule has a higher potential energy, it is less stable. Choice A is a trap since it indicates the more stable product, but the
more stable product may or may not form faster. The kinetic product will always form faster. While choice B may also sound
good because generally speaking, more substituted alkenes are more stable, the alkene is not the only part of the molecule that
may contribute to stability.
35 Biological Sciences Solutions
153. A. Nuclear magnetic resonance splitting patterns are determined by the n + 1 rule, where n is the number of Hs on the
adjacent carbon. Since the aldehyde proton has no neighboring Hs, we have 0 + 1 = 1, a singlet.
154. B. Washing with water will remove soluble impurities, not insoluble ones, so choice B (not C) is the answer. Choice A can
be eliminated since the product has already been recrystallized, and choice D is wrong because spreading the crystals evenly
over the surface of the funnel is irrelevant.
155. C. Carbonyl carbons have a double bond to the oxygen and two other single bonds, for a total of three electron groups
around the atom. Therefore, three hybrid orbitals are needed, so sp
2
is the best choice.
Passage IV (Questions 156160)
156. A. Excessive aldosterone would act on the kidneys, directly counteracting their ability to increase uid output. Aldosterone
causes an increase in the reabsorption of sodium and indirectly increases the reabsorption of water. This would prevent water
loss through the urine, increase blood volume, and keep blood pressure high. An excess of glucagon and a shortage of insulin
would both lead to excess glucose in the bloodstream, which would act osmotically to increase blood volume and therefore
blood pressure. However, the kidneys could still help by increasing urine output, as nothing is inhibiting that function under
these conditions (choices B and D are wrong). A shortage of thyroxine would most likely not affect blood pressure at all, but if
it did, it would probably lead to a decrease in blood pressure via a decrease in heart rate and general body metabolism (choice
C is wrong).
157. D. Since ltration is directly proportional to blood pressure, an increase in blood pressure necessarily produces an increase
in ltration rate and a subsequent increase in urine production. The bladder has nothing to do with urine production; it is simply
a storage tank for urine, and increasing its blood ow would have no effect on urine output (choice A is wrong). Increased
reabsorption of solutes and water would reduce the amount of water in the urine (reabsorption means to return a substance
to the bloodstream), not increase it (choice B is wrong). Likewise, increasing the permeability of the collecting duct to water
would result in increased water reabsorption and decreased urine output (note that this is the effect that ADHantidiuretic
hormonehas on the nephron, choice C is wrong).
158. C. Hypothesis B essentially says that increased blood pressure (the problem) leads to increase urine output and decreased
blood volume (one possible solution). Thus, the inverse must also be true: a decrease in blood pressure should lead to decreased
urine output and increased blood volume and pressure. The kidneys monitor pressure at the glomeruli because decreases in
glomerular pressure can reduce the rate of blood ltration, and this is potentially deadly. The application of clamps to the
renal arteries would result in a slight initial increase in blood pressure (due to the small increase in vascular resistance at these
arteries), but the lack of blood ow to the glomeruli would drop glomerular pressure and severely restrict ltration. In an effort
to overcome that problem, the kidneys would increase water reabsorption to boost blood volume and systemic blood pressure
in an effort to increase ltration (this is the sustained rise in blood pressure seen after a while). Thus, the drop in glomerular
blood pressure led to sustained hypertension. This is what is expected and therefore Hypothesis B is supported. Although the
clamps would lead to an initial increase in blood pressure, they cannot account for the sustained gradual increase (choice A is
wrong), and the fact that the kidneys are receiving less blood has nothing to do with Hypothesis A (choice B is wrong). There
is no reason to assume that the volume of body uids was decreasing (choice D is wrong).
159. A. Consider the equation in Hypothesis A: P = CO VR, which can be rewritten as P/VR = CO. The question states that
P doubled and VR increased by 50%, and wants to know what happened to CO. Since (2P)/(1.5VR) = (4P)/(3VR) = (4/3)CO,
we nd that CO increased by 1/3.
160. B. Hypothesis A discusses changes in vascular resistance, which is brought about by contraction and relaxation of vascular
smooth muscle. Striated muscle is skeletal muscle, and is multinucleate (choices A and D are wrong). Cardiac muscle is found
only in the heart (choice C is wrong).
MCAT Practice Test 8 Solutions 36
Free-Standing Questions (161 through 164)
161. C. Antibodies are secreted proteins and are therefore synthesized by ribosomes on the rough ER. The nucleus and
mitochondria have nothing to do with protein synthesis (choices A and B are wrong), and the Golgi apparatus is associated only
with packaging of secreted proteins, not their synthesis (choice D is wrong).
162. A. The knee-jerk reex is an example of a simple stretch reex and as such does not require the input of the brain
(choices B and C are wrong). Further, only one motor neuron is required to stimulate a skeletal muscle, not a series of motor
neurons (choice D is wrong).
163. D. In order to remove triethylamine from an organic solvent into the aqueous layer, you need to protonate the nitrogen and
give the molecule a charge. Therefore, treat with an acid.
164. C. The predominant conformation of the molecule will have the tert-butyl group in an equatorial position (since it is
so sterically large) in order to minimize 1,3-diaxial interactions. Eliminate choices A and B. Axial and equatorial positions
alternate as you move around the ring for substituents on the same face. Therefore, the chlorine must be axial.
Passage V (Questions 165169)
165. A. The only place in the nephron where ltration occurs is in the glomerulus. The loop of Henle establishes the medullary
concentration gradient and helps to reabsorb water (choice B is wrong), the distal convoluted tubule helps to ne tune urine
by reabsorbing necessary ions (for example, sodiumthis is where aldosterone has its effectchoice C is wrong), and the
proximal convoluted tubule is where most ltrate modicationreabsorption and secretionoccurs (choice D is wrong).
166. B. The passage states that colchicine is an inhibitor of microtubule reorganization, a process that is necessary for
phagocytosis. Preventing phagocytosis would decrease inammation and reduce pain. Diffusion through cell membranes is
not affected by microtubules (choice A is wrong), nor is uric acid crystal formation (choice C is wrong), nor is pH maintenance
(choice D is wrong).
167. C. The passage states that uric acid is formed by the breakdown of purines to xanthine. The only purine in the answer choices
is guanine. Cytosine, uracil, and thymine are all pyrimidines (choice C is correct, and choices A, B, and D are wrong.).
168. A. The passage states that the patient had normal levels of PRPP synthetase, so transcription and translation must not be
affected (choices C and D are wrong). Since the pH optimum and enzyme activity are normal when the enzyme is puried, the
active site must not be affected (choice B is wrong). The only time this enzyme malfunctions is when it is in the patient, where
its activity is stated to be three times normal levels. This indicates that there must be some other molecule in the patients cells
that is affecting the activity of this enzyme, and not by binding to the active site since product formation is high, but is otherwise
unaffected. Molecules that affect enzyme activity without binding to the active site are called allosteric regulators (choice A is
correct).
169. B. Flying birds must remain light; they have hollow bones and little body mass. Excess water would contribute to the
weight of the bird, so it would be an advantage for ying birds to excrete nitrogenous waste in the form of uric acid, instead of
ammonia or urea, which require water. Wild pigs typically inhabit areas where water is available (forests, as opposed to deserts)
so they can (and do) eliminate nitrogenous wastes as urea (choice A is wrong). Sharks and shes eliminate nitrogenous waste
as ammonia, which is highly toxic and must be diluted in large amounts of water. However, since they live in water, this is not
a problem (choices C and D are wrong).
37 Biological Sciences Solutions
Passage VI (Questions 170174)
170. A. The strength of an acid is related to the stability of its conjugate base. Since the negative charge on the nitrogen that
results from the proton abstraction can be stabilized by resonance with the adjacent carbonyl bond, A is the best answer.
Choice B can be eliminated since polarity does not necessarily correlate with acidity. Similarly, just because the amide is not
basic does not mean it is acidic. Eliminate choice D. An electron-donating aromatic ring would make the amide hydrogen less
likely to leave, and the functional group less acidic. However, because of the nitro group on the benzene ring, the substituent is
electron withdrawing. In either case, C is therefore incorrect.
171. C. As shown in Scheme 1 of the passage, the carbonyl of the amide group is lost in the Hofmann rearrangement, and a
new bond is formed between the alpha carbon and the nitrogen. Only choice C shows loss of the entire carbonyl while keeping
the nitrogen present in the molecule.
172. D. Hydrolysis of a carboxylic acid derivative yields the carboxylic acid itself. Choice A shows a reaction in which the
nitro group on the benzene ring is lost. This functional group will be unreactive towards base. Choice B is the product of the
Hofmann rearrangement shown in Scheme 1, so it is not the product of the competing reaction. Choice C is Compound 1, so
shows no reaction at all.
173. B. In this rearrangement, the reactant loses a carbonyl group, so the IR stretch that corresponds to the C=O bond would
disappear. Most C=O stretches appear close to 1700 cm
1
, so choice B is the best answer.
174. D. In this rearrangement, the reactant loses a carbonyl group, so the
13
C NMR signal that corresponds to the carbonyl
carbon would disappear. Although ketone carbonyls have signals around 200 ppm, carboxylic acids and their derivatives are
nearer 160170 ppm.
Passage VII (Questions 175180)
175. B. Typically, a triacylglycerol, or triglyceride, does not contain any stereocenters since two (or more) of the three fatty
acids are the same. However, if three different fatty acids are used, the second carbon of the glycerol becomes a stereocenter,
because four different substituents are attached.
176. D. Basic hydrolysis of an ester yields an alcohol (the glycerol) and a carboxylate anion (the functional group attached to
the fatty acid side chain). Only choice D shows a functional group with the two oxygens of the ester group still present in the
product.
177. D. As Reaction 1 shows, a triacylglycerol contains three fatty acid side chains. Eliminate choice A. Since the passage
states that the starting compound was a pure sample, it contained a maximum of three fatty acids. Therefore, some fraction of
one of the side chains must have been altered during the reaction. Choice C can be eliminated since complete conversion of one
fatty acid into another would yield a total of three fatty acids. An alcohol cannot be converted into a carboxylic acid under basic
conditions, so choice B can be eliminated.
178. C. An ester group contains a carbonyl attached to an OR groupin this case, the backbone of the glycerol molecule.
179. D. Saponication is not catalyzed by hydroxide since the base becomes incorporated into the carboxylic acid of the fatty
acid product. In other words, hydroxide is not regenerated in the reaction, so choice A can be eliminated. Each ester group
requires one equivalent of OH

to react, so for each molecule of triglyceride, three OH

ions are required. This means three


equivalents of base are necessary to completely saponify the reactant.
180. C. While organic molecules with charges are preferentially soluble in aqueous solution, answer choices A and B are too
clear cut. The charged end of a fatty acid salt helps make it soluble in a polar solvent, while the hydrocarbon end helps make it
soluble in a nonpolar solvent. The fatty acid salt is therefore amphipathic and can partially dissolve in both environments.
MCAT Practice Test 8 Solutions 38
Passage VIII (Questions 181186)
181. C. Testosterone is shown to inhibit the pituitary gland, turning off FSH, which would inhibit the Sertoli cells. In the
absence of Sertoli cell activity, sperm production is reduced. The passage states that the effect of testosterone on Sertoli cells
is to promote maturation of sperm, not to increase general activity such as release of inhibin, so there is no reason to assume
that inhibin concentrations would change (choices A and B are wrong). Testosterone would lead to a reduction in LH secretion
(also via inhibition of the pituitary gland), but this would simply lead to a reduction in natural testosterone production, not a
reduction in spermatogenesis (choice D is wrong).
182. D. The ovum is the most mature version of the female gamete. It corresponds with the most mature version of the
male gamete, the spermatozoon. The spermatogonium is the male version of the oogonium (choice A is wrong), the primary
spermatocyte is the male equivalent of the primary oocyte (choice B is wrong), and the spermatid is not a fully mature male
gamete (choice C is wrong).
183. C. Spermatogenesis is the only process in the list that requires cell division, so it would be the only process affected by
drugs that inhibit cellular proliferation (choice C is correct, and choices A, B, and D are wrong).
184. C. Statement I is false: LH is needed for testosterone production, not directly for sperm production. Statement II is true:
FSH is needed to stimulate Sertoli cells, which are needed to promote spermatogenesis. Statement III is false: Inhibin turns off
FSH secretion and Sertoli cell activity. Statement IV is true: Testosterone is needed to promote Sertoli cell activity.
185. A. As nurse cells, Sertoli cells secrete nutrients, and the Golgi apparatus is necessary for cellular secretion. The
lysosomes break down ingested nutrients and old cellular organelles (choice B is wrong), the mitochondria produce ATP for the
cell (choice C is wrong), and cilia are used to sweep material along the surface of the cell (choice D is wrong).
186. B. This is the basic difference between exocrine and endocrine tissue: the location of their secreted products. The passage
describes the exocrine portion of the testes as sperm production (note that sperm are not peptides, choice A is wrong), which
takes place inside coiled ducts called seminiferous tubules. The endocrine portion produces testosterone, a hormone that is
released into the blood. The Sertoli cells of the seminiferous tubules (part of the exocrine portion) secrete nutrients, so the
exocrine portion does not only secrete cellular elements (choice C is wrong). The exocrine portion is only one of several targets
for the product of the endocrine portion; furthermore, it is the target for many other products as well (choice D is wrong).
Free-Standing Questions (187 through 191)
187. D. As far as its digestive role goes, the pancreas produces digestive enzymes (including proteases) and bicarbonate,
which it releases into the small intestine. (Note that the pancreas also produces insulin and glucagon, but this is not part of its
digestive function.) Bile salts, which are emulsiers, are produced in the liver (choices A and B are wrong), and gastric juices,
by denition, are produced by and released into the stomach (choice C is wrong).
188. C. Mitochondria are only found in eukaryotes. Cell walls and ribosomes are found in both prokaryotes and eukaryotes
(choices A and B are wrong), and both prokaryotes and eukaryotes exhibit sexual reproduction (choice D is wrong).
189. A. The products of acidic hydrolysis of an ester are an alcohol and a carboxylic acid. The water acts as a nucleophile,
attacking the ester carbonyl, and the oxygen from the water is incorporated into the COOH group. The OR group of the ester,
in this case the cyclopentyloxide, becomes the leaving group (choice D). Choices B and C are molecules that are not involved
as reactants or products in this additionelimination reaction.
190. A. Statement I is false: Ectoderm ultimately forms the skin, hair, nails, and nervous system. Statement II is true: Mesoderm
forms muscles, bones, blood vessels, and the heart, as well as non-glandular organs, such as the kidneys and the walls of the
digestive tract, bladder, and uterus. Statement III is false: Endoderm primarily forms the linings of the digestive system, the
respiratory system, and the reproductive system. (Note also though, that it forms the linings of the cardiovascular system, such
as the chambers of the heart and the inner linings of the blood vessels, but most of the heart and vessels structure is formed from
mesoderm, making choice A a better answer than choice B.)
39 Biological Sciences Solutions
191. D. Most of the water lost through the skin is lost as sweat, so raising the environmental temperature would lead to an
increase in sweating and an increase in water loss. Inhibiting kidney function would prevent water loss through urination, and
might promote an increased loss through the skin, but this would be strongly dependent on temperature as well (choice A is
wrong). Decreasing salt consumption and increasing water consumption would both have the effect of increasing urinary water
loss, but would not affect water loss through the skin (choices B and C are wrong).
Passage IX (Questions 192197)
192. B. The passage states that the dewlap is part of a visual communication system that involves reection of UV rays. UV
receptors were found in the eyes of lizards involved in this type of communication. The signicance of UV would be most
easily determined by comparing the behavior of responding lizards that are exposed to reected UV rays with the behavior
of lizards that are not exposed to reected UV rays (because the dewlap of the ashing lizard has been treated to absorb UV).
Comparing sighted and sightless lizards might give information about the signicance of visual communication in general,
but would not give specic information regarding UV reection (choice A is wrong). Observing all ve lizard species in the
same habitat might give information about the response to the amount of UV reected, but again, would not give information
about the signicance of reecting UV rays in the rst place (choice C is wrong). Lastly, observing the species under red light
illumination would give no information about UV signicance at all, since red light is at the opposite end of the spectrum from
UV (choice D is wrong).
193. D. Since species E lives in the shaded canopy of the forest, and since it has the lowest reection of UV radiation by its
dewlap, it would probably suffer least from a mutation that prevents detection of UV rays by the eyes. Species A and B both
live in the open unshaded elds where plenty of UV radiation is found, so they probably rely more strongly on UV detection
(choices A and B are wrong). Lastly, species D, although it also lives in the shaded canopy, reects more UV rays with its
dewlap than does species E, indicating that UV radiation is probably more important to species D than to species E (choice C
is wrong).
194. D. Simply knowing that Anolis lizards have X-Y chromosomal sex determination does not give us any further information
about the gene for the UV reectance pigment and where it is found. If UV reectance were strictly seen in males, the gene for
this pigment might be found on the Y chromosome, but this is not a absolute guarantee. In any case, we dont know if females
also have this pigment and the expression of that gene is simply downregulated, or if its a recessive gene and is carried on the
X chromosome (meaning males are more likely to express it). Bottom line: just because a gene is expressed in males, do not
automatically assume that the gene must be located on the Y chromosome (choice D is correct, and choices A, B, and C are
wrong).
195. D. The denition of separate species is that they cannot breed and produce fertile offspring with each other (or with
members of any other species, for that matter). The same species can occupy different habitats (owls can live in barns, elds, or
forests, choice A is wrong), can have different physical characteristics (some tigers are white and some are orange, choice B is
wrong), and may not communicate with one another (for example, it would be difcult for a eld mouse in the United States to
communicate with a eld mouse in Europe even if they were the same species, choice C is wrong). However, if two populations
cannot breed or produce fertile offspring, then they are separate species.
196. B. Figure 1 shows that dewlap reectance is correlated with habitat. All three species that live in unshaded elds have
high reectance, while the two species that live in the shaded canopy have low reectance (choice B is correct, and choice C is
wrong). Note, however, that the correlation does not mean that habitat is determined by dewlap reectance (choice A is wrong).
Figure 1 also shows that dewlap reectance is actually quite low at the visible blue frequencies, and in some species (E for
example) is highest in the green-yellow frequencies (choice D is wrong).
197. A. Evolution is driven by benecial characteristics that lead to the production of greater numbers of surviving offspring.
A characteristic can be benecial (better communication, less predation, more frequent mating), but if it does not result in
greater numbers of surviving offspring, then the characteristic will not be increased by natural selection (choices B, C, and D
are wrong).
MCAT Practice Test 8 Solutions 40
Passage X (Questions 198204)
198. C. At concentrations of actin greater than 1 M, the rate of subunit addition to the + end is greater than the rate of addition
at the end. At exactly 1 M the + end is not growing at all and the end is losing subunits. Under these conditions the
microlament would be shrinking at the end (choice A is wrong). At concentrations between 1 M and 4 M, the microlament
is denitely growing, but it doesnt stop this at concentrations greater than 4 M (choice B is wrong). At concentrations lower
than 1 M, both the + and the ends are losing subunits, so the microlament is shrinking overall (choice D is wrong).
199. B. If both ends of the microlament are capped, no growth is possible. If the ends are capped by the Z lines and the actin
concentration is greater than 1 M, then the uncapped + ends would continue to grow and the microlaments would get longer
(choice A is wrong). If the actin concentration were kept above 4 M, then both the + and ends would be growing (choice
C is wrong). If the ends grow at the same rate that the + ends shrink, then the microlament is treadmilling but is not stable
(choice D is wrong).
200. A. The theory of force generation states that microlaments can generate force by growing and pushing against the
plasma membrane of a cell. Cytochalasin prevents growth of the microlaments and, if the theory is correct, should stop the
pushing of the plasma membrane that is seen in pseudopod formation (that is, amoeboid movement). Mitosis involves the use
of microtubules, not microlaments, so blocking mitosis and movement would not support the theory of force generation by
microlaments (choice B is wrong). Troponin does not play a role in the theory of force generation (choice C is wrong), nor is
anything stated about the viscosity of the medium (choice D is wrong).

201. C. Figure 1 shows that when the actin concentration is 1.5 M, the rate of addition of subunits to the + end is exactly equal
to the rate of removal of subunits from the end, and this is dened in the passage as treadmilling. At 0.25 M both ends
are losing subunits (choice A is wrong), at 1.0 M the + end is stable but the end is losing subunits (choice B is wrong), and
at concentrations between 1 M and 4 M, the + end is gaining subunits while the end is losing them; however, the rate of
addition is much greater than the rate of loss, and the microlament would be growing toward the + end (choice D is wrong).
202. A. Any actin concentration below 1 M results in a loss of subunits from both ends. At exactly 1.0 M the end is losing
subunits, but the + end is stable. Note that this is a net loss, but that the loss of subunits at concentrations below 1 M is much
greater because both ends are losing subunits (choice A is better than choice B). At concentrations above 1 M, the rates of
loss and gain vary, and in some cases both ends are gaining subunits (choice C is wrong). At concentrations between 1 M and
4 M, the + end is gaining subunits while the end is losing them; however, the rate of addition is much greater than the rate
of loss, and the microlament would be growing toward the + end (choice D is wrong).
203. A. If exocytosis of viruses and microlament polymerization are proportional, then they could be linked. If treatment with
phalloidin, which prevents microlament growth, does not stop the release of viruses, then the two processes are independent
(choice B is wrong). Viruses would not need to carry genes for actin subunits, since those would be coded for by the host cell
(choice C is wrong). Capsules made of myosin might support the use of microlaments in viral release, but linking the rates of
exocytosis and microlament polymerization (as in choice A) is better evidence (choice D is wrong).
204. D. At actin concentrations of 2 M, the + end on the right is adding subunits faster than the end on the left is losing
them, causing the microlament to grow towards the right. This would push the Amoeba towards the right. At 0.5 M both the
+ end and the end would lose subunits, and the Amoeba would be going nowhere (choice A is wrong). At 6 M both the +
end and the end would be gaining subunits, and the Amoeba would be stretching in both directions (choice B is wrong). At
1 M the + end would be stable and the end would be losing subunits, and the Amoeba would be shrinking at the left (choice
C is wrong).
Passage XI (Questions 205208)
205. A. Equation 1 in the passage shows the migration of a methyl group, an example of a rearrangement. In order for a skeletal
rearrangement to occur, a carbocation intermediate must be produced. Only the addition of a proton from the H
2
SO
4
to the
alkene yields such an intermediate.
41 Biological Sciences Solutions
206. C. Equation 3 in the passage shows the addition of the OH group to the least substituted carbon of the double bond.
This corresponds to the outcome of the reaction in the question. Both choices A and B would add the OH group to the most
substituted carbon of the starting material, while the passage indicates nothing about heat accomplishing a hydration reaction
with no other reagents present.
207. A. Molecules containing OH groups, as alcohols do, can hydrogen bond. Hydrogen bonds are the strongest of the
intermolecular interactions (stronger than van der Waals forces in choice B). This means they have strong attractions, requiring
more energy to vaporize them. Choices C and D relate to intramolecular interactions, which do not determine physical properties
of molecules such as boiling points.
208. A. Grignard reagents are added to carbonyl compounds to produce alcohols as products. Therefore, choices B and C
can be eliminated. In the presence of alcohols, Grignard reagents will act as bases and deprotonate the OH group. In order to
form a tertiary alcohol product, a ketone starting material is required. The aldehyde in choice D will yield a secondary alcohol
product.
Free-Standing Questions (209 through 214)
209. A. Watson and Crick proposed the complementary theory of base pairing, A with T and G with C. The nding that the
amounts of these bases were proportional supported their theory. This does not give any evidence for DNA being genetic
material (choice B is wrong), for the genetic code being the same in all organisms (choice C is wrong), or for a triplet genetic
code (choice D is wrong).
210. B. The number of different gametes that can be produced is equal to 2
n
, where n is the number of heterozygous alleles
the organism possesses. In this case, the organisms genotype is AaBbCc. Since it possesses 3 heterozygous pairs of alleles, the
number of different gametes it can produce is 2
3
= 8.
211. C. If the tidal volume (amount of air moved in a single breath) is 800 mL and the dead space volume is 150 mL, then the
amount of air reaching the alveoli is only 650 mL. If the breathing rate is 10 breaths/min, then the amount of air reaching the
alveoli is 650 mL/breath 10 breaths/min = 6500 mL/min.
212. B. Splitting of centromeres (separation of sister chromatids) happens in mitosis and in meiosis II, but not in meiosis I.
Synapsis (pairing of homologs) occurs only in meiosis I (choices A and C are wrong). Breaking down of the nuclear membrane
occurs in both mitosis and meiosis I (choice D is wrong).
213. D. The tall pink plant heterozygous for height has the genotype TtRr. The short pink plant has the genotype ttRr. The
probability of getting a pink plant (a heterozygote) by crossing two heterozygotes is 1/2, and the probability of getting a tall
plant by crossing Tt and tt is also 1/2. Thus, using the Rule of Multiplication, to get a plant that is pink and tall is 1/2 1/2 =
1/4.
214. D. Eukaryotes transcribe RNA in the nucleus and translate proteins in the cytosol, so the processes cannot occur
simultaneously. Both eukaryotes and prokaryotes use ribosomes for protein translation (choice A is wrong), and both use
DNA to transcribe mRNA (choice B is wrong). The rate of destruction of mRNA varies depending on the mRNA (choice C is
wrong).

Das könnte Ihnen auch gefallen